You are on page 1of 93

HIGH-YIELD INTERNAL MEDICINE BOARD EXAM PEARLS

This study guide includes:


Chapters covering High-Yield pearls for all major categories seen on the Internal
Medicine Board Exam / Shelf Exam
Topics covered include Cardiovascular Disease, Endocrinology & Metabolism,
Gastroenterology, General Internal Medicine, Hematology, Infectious Disease,
Nephrology / Urology, Oncology, Neurology, Dermatology, Pulmonary Disease &
Critical Care, Rheumatology / Orthopedics

www.knowmedge.com

ABIM is a registered trademark of the American Board of Internal Medicine, which neither sponsors nor is affiliated in any way with this product.

KNOWMEDGE

HIGH-YIELD INTERNAL MEDICINE BOARD EXAM PEARLS

Email: support@knowmedge.com
Website: www.knowmedge.com
Facebook: www.facebook.com/knowmedge
Twitter: www.twitter.com/knowmedge
YouTube: www.youtube.com/knowmedge
The content in this book may be updated periodically. All updates, including known errors or revisions, are available at:
www.knowmedge.com/forum
Copyright 2013 Knowmedge
ALL RIGHTS RESERVED. No part of this work may be reproduced or used in any form or by any meansgraphic, electronic, or
mechanical, including photocopying, recording, taping, web distributionwithout the prior written permission of Knowmedge
Note: ABIM is a registered trademark of the American Board of Internal Medicine, which neither sponsors nor is affiliated
in any way with this product.

Dear Reader,
Thank you for downloading a copy of this eBook. If you are searching for study materials for the internal
medicine board exam, you are on the verge of a significant milestone in your professional journey.
Knowmedge is an interactive platform that was launched in April 2013. It features over 900 questions
designed to help you understand and reinforce the key concepts covered on the exam. Each of our questions
features a highly interactive audio visual explanation, in which our content experts walk you through the
principles underlying each question to methodically arrive at the correct answer.
This book contains high-yield pearls for the Internal Medicine Boards ABIM Exam and IM Shelf Exam
written by the team of Knowmedge doctors. There are a total of 12 different pearl articles presented in
this book all from topics that are important to pass the Internal Medicine Boards.
We hope you find the pearls in this book to be a valuable asset as you prepare for your upcoming exam. If
you are interested in learning more about Knowmedge, please visit us at www.knowmedge.com.
If you have any questions about the contents of this eBook, send me a note at sunir@knowmedge.com
Best of luck in your preparations!
Sincerely,

Sunir
Sunir Kumar, MD
Co-founder, Chief Editor
Knowmedge

Table of Contents
5 Cardiology Pearls .................................................................. 1
By: Dr. Salim Rezaie

5 Endocrinology Pearls ............................................................ 8


By: Dr. Sunir Kumar

5 Gastroenterology Pearls ...................................................... 12


By Dr. Sunir Kumar

Gastroenterology Pearls: Digestive Tract in 7 Major Parts ..... 19


By: Dr. Ruchi Bhatia

General Internal Medicine Pearls: Eye Conditions ................. 25


By: Dr. Sunir Kumar

General Internal Medicine Pearls: Vaccinations ..................... 29


By: Dr. Sunir Kumar

5 Hematology Pearls .............................................................. 32


By Dr. Salim Rezaie

6 Infectious Disease Pearls .................................................... 36


By Dr. Salim Rezaie

11 Nephrology Pearls ............................................................. 40


By Dr. Salim Rezaie

7 Oncology Pearls .................................................................. 45


By: Dr. Sunir Kumar

5 Neurology Pearls ................................................................. 51


By: Dr. Sunir Kumar

Quick and Easy Guide to Selecting the Right Topical Steroid 57


By: Dr. Sheila Krishna

8 Pulmonary Disease & Critical Care Pearls .......................... 61


By: Dr. Sunir Kumar

5 Rheumatology Pearls .......................................................... 68


By: Dr. Sunir Kumar

Bonus: How to study for and pass the ABIM board exam ....... 72
By: Dr. Ravi Bhatia

Bonus: How to study for the Internal Medicine Shelf Exam .... 78
By: Dr. Ravi Bhatia

About our Authors .................................................................. 87

K N O W M E D G E

5 Cardiology Pearls
By: Dr. Salim Rezaie
Whether studying for emergency medicine, internal medicine, or USMLE board examinations,
cardiovascular diseases are by far and away one of the biggest organ systems of which
questions get asked. If you pay attention, the first three pearls are all on physical exam findings
and the last two pearls are on disease processes with high morbidity and mortality. I have
attached a review table for each pearl to help simplify the concepts and buzz phrases.
Pearl #1: Know what the maneuvers are that increase and decrease heart murmurs.
In general, you should know all the different maneuvers, their effect, and how they would affect
valve abnormalities. It is helpful to walk yourself through each valve abnormality, and try and
explain why murmurs are increased or decreased, instead of just memorizing a table, which
you will forget in 1 2 weeks. That being said, common murmurs that seem to get the most
questions are: HOCM, AS, MVP, and MR.

Cardiac Maneuvers for the Internal Medicine Board Exam


1|Page

K N O W M E D G E

Pearl #2: Know the abnormal arterial pulsations and the disease state with which they
are commonly associated.
Arterial pulsations is another physical exam finding that can be very high-yield. Recognizing
the buzz phrase (arterial pulse description) and its association to what disorder it is commonly
associated can help save you time on questions, which allows you more time on other
questions. This is too easy to not know these terms.

Heart Pulses

Pearl #3: Know your normal and abnormal heart sounds.


2|Page

K N O W M E D G E

Heart sounds such as S1 and S2 are also a big part of the cardiovascular physical exam on
boards. Know what happens with inspiration/expiration as well as other pathologies. Dont
forget about S3 and S4. Remember, sometimes an S3 can be normal (i.e. pregnancy and
children).

Heart Sounds to know for the Internal Medicine Boards

Pearl #4: The number one killer in the world is ischemic heart disease, so know what
medications improve morbidity and mortality.
3|Page

K N O W M E D G E

First and foremost, be sure to fix modifiable coronary artery disease risk factors such as
diabetes mellitus, hypertension, hyperlipidemia, and smoking. Next, know what medications
decrease mortality in ischemic heart disease and acute coronary syndrome. Also be sure to
know the contraindications for thrombolytics in STEMI.

CAD ACS Therapy Table for the Internal Medicine Boards Part I

4|Page

K N O W M E D G E

CAD ACS Therapy Table for the Internal Medicine Boards Part II

5|Page

K N O W M E D G E

Contraindications Thrombolytics
Pearl #5: Congestive Heart Failure (CHF) is common, so know which medications affect
mortality
CHF is a common disease process seen in hospitalized, elderly patients due to improvements
in revascularization techniques. 40% of these patients die from arrhythmias and sudden
cardiac death; therefore knowing what medications can improve mortality is essential.

6|Page

K N O W M E D G E

CHF Medications

7|Page

K N O W M E D G E

5 Endocrinology Pearls
By: Dr. Sunir Kumar
Endocrinology is an essential part of the Internal Medicine Medical Clerkship and ABIM Board
exam. According to the ABIM exam blueprint, questions testing endocrinology topics comprise
~8% of the exam. Approximately ~5-10% of the NBME Clerkship exam is composed of
endocrinology questions.
Pearl #1: Dont let thyroid nodules intimidate you.
This systematic approach will help you workup a thyroid nodule.

8|Page

K N O W M E D G E

Pearl # 2: Workup of hirsutism is not as difficult as it seems. Follow this approach and
you will be able to diagnose the cause of hirsutism.

Hirsutism is caused by either excessive testosterone or excessive 17-OH steroids


(DHEA-S) production.
Excessive Testosterone production is seen in ovarian cancer or polycystic ovarian
syndrome (PCOS)
o Ovarian cancer: worked up with trans-vaginal ultrasound to look for adnexal
mass. In addition, CA-125 marker is usually elevated in ovarian cancer.
PCOS: Amenorrhea, insulin resistance, and LH:FSH ratio of greater than equal
to 3:1
Excessive DHEA-S production is seen in congenital adrenal hyperplasia (CAH),
Cushings disease, or adrenal carcinoma
CAH: Usual cause is 21 beta hydroxylase deficiency, which is used to convert 17-OH
progesterone to cortisol. Since this enzyme is deficient, 17-OH progesterone levels
remain elevated. Decreased cortisol levels will cause an elevated ACTH level through
a negative feedback mechanism. With increased ACTH, hyperpigmentation will also
occur.
Cushings disease: Defect in anterior pituitary causes increased ACTH, which
o

increases cortisol. MRI of pituitary is ordered to work up Cushings. If suspicion is high


for Cushings disease despite negative MRI of the pituitary, perform inferior petrosal
sinus sampling.
Adrenal carcinoma: Problem occurs in the adrenal gland, which will lead to elevated
cortisol levels. The elevated cortisol level will suppress the ACTH level. Since ACTH is
suppressed, hyperpigmentation will not occur.

Pearl # 3: Workup of an adrenal mass and management depends on the size and the
functional status of the mass

Many times, adrenal masses are found incidentally on a CT scan. These are known as
an adrenal incidentalomas.
Rules to remember:
o If the adrenal mass is either greater than 6 cm in size OR is functional
(regardless of size) surgical intervention is recommended
9|Page

K N O W M E D G E

If an adrenal mass is less than 4 cm AND is non-functional serial CT scans


are recommended every 4-6 months to assess the size of the adrenal mass to
make sure it is not growing
How to determine functional status of an adrenal mass? Remember the 3 layers
of the adrenal cortex and the one layer of the adrenal medulla and know what is
produced in each layer to determine if it is functioning or not.
Adrenal Cortex layers (remember by mnemonic GFR as in glomerular filtration
rate):
Zona Glomerulosa check to see if aldosterone:renin ratio is elevated
(usually more than 20:1)

Zona Fasciculata check 24 hour urine cortisol levels and if greater than
100 mg/dl Cushing problem should be suspected
Zona Reticularis Check 17 OH steroid (DHEA-S) levels. If elevated,
this layer is functioning.
Adrenal Medulla: Check urine VMA or urine metanephrine levels. If either
of these metabolites are elevated, concern is for pheochromocytoma

Pearl # 4: Systematic approach to workup hypercortisolism

First check 24 hour urine cortisol. If greater than 100 mg/dl, then you either have
Cushing syndrome, Cushings disease, or ectopic production of ACTH.
Next step is to check the ACTH level. If the ACTH level is suppressed, then the problem
is Cushing syndrome; CT or MRI of adrenals should be done. If ACTH level is elevated,
the patient has either Cushings disease (pituitary problem) or ectopic production of
ACTH (like lung cancer)
To distinguish between Cushings disease and ectopic production of ACTH, perform a
high dose (8mg) dexamethasone suppression test.
o If high dose dexamethasone suppresses cortisol, problem is Cushings disease.
MRI of the pituitary should be performed. If MRI of the pituitary is negative,
o

perform inferior petrosal sinus sampling.


If high dose dexamethasone suppression test fails to suppress cortisol, the
problem is ectopic production of ACTH. Check CT scan of chest to rule out lung
cancer.

Pearl # 5: Must know diabetes mellitus high yield facts


10 | P a g e

K N O W M E D G E

Type 1 DM lack of insulin because of destruction of pancreatic beta cells


associated with antibodies to glutamic acid decarboxylase
Type 2 DM more common in obese individuals and can occur later in life. Insulin
resistance occurs.
Diagnose of DM is made when patient has two fasting glucose levels greater than or
equal to 126 mg/dl or a random glucose level greater than 200 mg/dl especially in the
context of signs and symptoms like polyuria, polydipsia, or unintentional weight loss.
Goal Hgba1C is less than 7%. Hgba1C is an average glucose in a 3 month period.
Pre-prandial glucose goal in a DM patient is 90-130 mg/dl. 2 hour post-prandial glucose
goal is less than 180 mg/dl.

Monofilament foot testing is the best way to prevent diabetic foot ulcers from occurring.
A common organism that causes diabetic foot ulcers is Staph aureus or beta hemolytic
streptococcus.
Eye exams in DM patients are recommended every 1 to 2 years
o If eye exam reveals hard exudates or microaneurysms patient has nonproliferative retinopathy management is by tighter glucose control
o If eye exam reveals neovascularization or cotton-wool spots patient has
proliferative retinopathy treat with photocoagulation

Once again, the folks who write the Internal Medicine licensing exams dont expect you to
have the depth of knowledge regarding hormone-related conditions, metabolism and diabetes
that an endocrinologist possesses. However, topics such as the ones mentioned in the pearls
above should assist you with the endocrinology section of the med school clerkship shelf and
ABIM board exams.

11 | P a g e

K N O W M E D G E

5 Gastroenterology Pearls
By Dr. Sunir Kumar
Gastroenterology and Hepatology comprises about 9% of the ABIM Internal Medicine exam,
making it one of the more critical subjects on the boards. Below, we review 5 High Yield
Gastroenterology / Hepatology Pearls that may help you score a few extra points on your
ABIM or Internal Medicine shelf examination.
Pearl # 1: Remembering Hepatitis B markers can be difficult, but is worth it
Start with these key points:
o

Hepatitis B surface Antigen (HBsAg) active infection

Hepatitis B surface Antibody (HBsAb) past infection or vaccination against


hepatitis B
Hepatitis Be Antigen (HBeAg) active replication of the virus
Anti Hepatitis B core IgM Antibody (Anti-HBc IgM) acute infection
Anti Hepatitis B core IgG Antibody (Anti-HBc IgG) chronic infection

o
o
o

12 | P a g e

K N O W M E D G E

As you see above: Along with the Cleared state, both Chronic and Carrier Hepatitis B patients
will have positive HBsAg and Anti-HBcIgG. How can these two conditions be differentiated?
Easily. Just look at the Liver function tests (LFTs)
o
o

Chronic Increased LFTs


Carrier Normal LFTs

What about that Hepatitis D virus?


o

Hepatitis D cant exist on its own. It requires Hepatitis B infection to be present

Anti-HBc IgM + Hepatitis D virus acute co-infection and will not worsen
hepatitis
Anti-HBc IgG + Hepatitis D virus acute super-infection and can cause
fulminant hepatitis

13 | P a g e

K N O W M E D G E

Pearl # 2: Main causes of dysphagia can be broken down into Mechanical & Motility
Problems
Mechanical Problems (dysphagia to solids first and liquids later)

Intermittent problem
Esophageal ring (a.k.a Steakhouse Syndrome)
Clue in history: Patient has difficulty swallowing while
chewing foods like bread or steak.
Long standing history of GERD
Barretts esophagus (squamous to columnar metaplasia) or
stricture formation.
Dysphagia to solids with significant weight loss
If long standing smoking history, Squamous cell carcinoma
more likely
If long standing history of uncontrolled GERD,
Adenocarcinoma more likely
Eosinophilic esophagitis
Dense eosinophilic infiltrate in the squamous epithelium

Mainstay of treatment is viscous budesonide, fluticasone,


or proton pump inhibitors (PPIs)

Motility Problems (dysphagia to both solids and liquids concurrently)

Intermittent and associated with chest pain, especially after drinking


carbonated drinks
Diffuse Esophageal Spasm (DES)
Associated with connective tissue disease or heartburn
Scleroderma
Associated with cough/regurgitation with improvement of dysphagia with
raising hand above the head
Achalasia (which can in the context of Chagas Disease)
Before treating with surgical myotomy or pneumatic dilatation,
EGD must be done first to rule out secondary achalasia from
lymphoma or cancer.
14 | P a g e

K N O W M E D G E

Achalasia will have increased LES tone on manometry studies.


GERD, on the other hand, will have decreased LES tone.

Pearl # 3: Remember the main causes of Pancreatitis by the mnemonic I GET


SMASHED

Pearl # 4: Alcoholics arent the only folks to develop cirrhosis


To the lay public, cirrhosis is to alcohol as lung cancer is to smoking. However, we
know that its not such a simple association. Smokers arent the only patients to develop
lung cancer and those who dont drink alcohol can still become cirrhotic. Lets review
some of the non-alcohol related causes of liver failure with two easy-to-digest slides:
15 | P a g e

K N O W M E D G E

First, we review Viral Hepatitis, Fatty Liver/Steatohepatitis, Wilsons, Alpha 1 AntiTrypsin Deficiency, Hemochromatosis and Budd-Chiari along with helpful clinical clues
that may appear in the question vignette:

Of course, we cant forget Autoimmune Hepatitis, Primary Biliary Cirrhosis, and Primary
Sclerosing Cholangitis. Many medical students and residents find it confusing to match up the
gender, age, and serologies with the correct condition. While these are not hard-and-fast rules,
for exam purposes, in general we can use the following colorful schematic to make it tough to
ever forget again.

16 | P a g e

K N O W M E D G E

Pearl # 5: They may both be considered IBD, but know how to distinguish Ulcerative
Colitis and Crohns Disease
Ulcerative Colitis
o

Presents with abdominal pain and bloody diarrhea that is chronic

Problem starts in the rectum and spreads proximally (backwash ileitis)


Pathology reveals superficial ulcers with crypt abscesses
Some common extra-intestinal manifestations include:
Aphthous ulcers
Pyoderma gangrenosum
Primary sclerosing cholangitis

o
o

17 | P a g e

K N O W M E D G E

o
o

Ankylosing spondylitis
Arthritis (mirrors Ulcerative Colitis)
Erythema nodosum (mirrors Ulcerative Colitis)
Can show positive p-ANCA on labwork
Colonoscopy should be performed 8 years after diagnosis and then every 1-2
years afterwards as patients with UC have an increased risk of colon cancer
If dysplasia is seen on colonoscopy, total proctocolectomy should be
performed

Crohns Disease
o

o
o

o
o

Rectum is spared with Crohns Disease. Crohns Disease occurs mainly in the
terminal ileum (Skip lesions are common)
Common presentation is right lower quadrant mass, weight loss, and diarrhea
(can be bloody but can also be watery)
Pathology will show deep ulcers with granuloma formation
Not as common to see extra-intestinal manifestations with Crohns Disease as
is seen in patient with UC
Colon cancer can occur but is more common in UC patients
Can show positive ASCA (anti-Saccharomyces Cerevisiae antibodies) on
labwork
Colonoscopy should be performed 8 years after diagnosis and then every 1-2
years
If dysplasia is seen on colonoscopy, total protocolectomy should be
performed

18 | P a g e

K N O W M E D G E

Gastroenterology Pearls: Digestive Tract in 7 Major Parts


By: Dr. Ruchi Bhatia
As previously mentioned, the Gastroenterology and Hepatology section of the ABIM Internal
Medicine exam comprises of about 9% of the entire exam - that means out of 240 total
questions (4 sections of 60 questions each), we can expect about 20 questions to be geared
towards our liver and GI tract. For the Internal Medicine Shelf Exam, Gastroenterology
comprises 7-13% of the exam.
Overall, the digestive system is fascinating (the liver itself is the largest organ in the body and
performs over 500 functions!) yet quite simple (think of it this way food goes in to the mouth,
down the esophagus and in to the stomach, through 26 feet of small intestine in to the colon.
Then out.)
To simplify for the ABIM exam, lets divide the digestive tract in to 7 major parts and discuss
a couple important topics in each Esophagus, Stomach, Pancreas, Biliary Tract, Small
Bowel, Colon, and Liver.
1. Esophagus
A favorite topic of boards is GERD and the development of Barrett esophagus. GERD
is caused by a decrease in the physiologic antireflux barriers at the GE junction,
resulting in gastric contents being released in to the esophagus. Surprisingly, the major
cause of GERD is not hypersecretion of gastric contents, but rather an inappropriate
relaxation of the lower esophageal sphincter.
Remember:
o
o

A 4-week empiric trial of a PPI has a high sensitivity for the diagnosis of GERD
Patients presenting with weight loss, dysphagia, odynophagia, or those whose
symptoms are refractory to medical therapy should undergo further testing
Ambulatory esophageal pH monitoring is the most accurate means to confirm
the diagnosis of GERD

The development of Barrett esophagus is a feared complication of GERD due to the


increased risk for esophageal adenocarcinoma (remember, squamous cell carcinoma
19 | P a g e

K N O W M E D G E

arises in the upper portion of the esophagus and adenocarcinoma arises distally, closer
to the GE junction).
Remember:
o

Histologically, Barrett esophagus has specialized intestinal metaplasia with


mucin containing goblet cells
Dysplasia found during EGD:
none -> surveillance EGD should be repeated in 1 year, then every 5
years if negative

low grade -> surveillance in 6 months for 1 year, then yearly


high grade -> surveillance every 3 months for focal dysplasia vs. surgical
or endoscopic management for multifocal dysplasia

2. Stomach
Given the large number of hospitalizations and deaths from peptic ulcer disease every
year, it is not surprising that this remains a major topic tested on the boards. The most
common causes are Helicobacter pylori infection and NSAIDs.
Remember:

H. pylori is associated with the development of gastric adenocarcinoma

as well as MALT (mucosa-associated lymphoid tissue) lymphoma


Treatment for H. pylori consists of triple therapy PPI, Amoxicillin, and

Clarithromycin (Metronidazole in Clarithromycin resistant areas)


Urea breath test and fecal antigen test are both sensitive for the detection
of H. pylori

Be able to recognize a patient with a perforated peptic ulcer! Look for a patient who is
hypotensive and tachycardic with absent bowel sounds and severe rebound
tenderness and guarding. Imaging will show free intraperitoneal air. Stat surgical
consultation is required!
3. Pancreas

20 | P a g e

K N O W M E D G E

The prevalence of pancreatitis continues to rise in the Western world, and thus remains
a favorite for the ABIM boards. Although alcohol and gallstones remain the major cause
of acute pancreatitis, metabolic (hyperlipidemia, hypercalcemia), infectious (CMV,
EBV, parasites), and autoimmune causes should be considered.
Remember:

Diagnosis of acute pancreatitis can be made without imaging, but


contrast-enhanced CT scan is used if there is concern for necrotizing
pancreatitis

If necrotizing pancreatitis is suspected, prophylactic antibiotics should be


used imipenem, cephalosporins, and fluoroquinolones
ERCP is used if there is evidence of gallstone pancreatitis and suspected
biliary obstruction
Consider a deficiency in fat-soluble vitamins (A, D, E, K) in chronic
pancreatitis

Look for CA 19-9 as a tumor marker for pancreatic cancer. Better yet, be able to
recognize the whole table of important tumor markers discussed in the oncology section
of Knowmedge.
4. Biliary Tract
The prevalence of gallstones is high in the United States, and thus should be
considered as part of a differential for a patient presenting with abdominal pain. Be able
to recognize and know how to treat acute cholecystitis, but also know when to expect
and how to treat acalculous cholecystitis.
Remember:

Consider acalculous cholecystitis in patients with serious comorbidities,


including trauma, burns, or prolonged states of fasting
Management is similar to that of acute calculous cholecystitis, but
patients with severe illness may require percutaneous drainage if unable
to tolerate surgery
21 | P a g e

K N O W M E D G E

If choledocholithiasis is suspected, broad-spectrum antibiotics covering enteric gramnegative bacteria should be started. Fluoroquinolones are usually a good initial choice.
5. Small Bowel
Another favorite boards topic diarrhea. Diarrhea can of course be divided multiple
ways acute vs. chronic, secretory vs. osmotic vs. inflammatory, small-bowel vs. largebowel. Be able to easily distinguish the two main types of inflammatory diarrhea,
Ulcerative colitis vs. Crohns. Since this was already discussed in the last GI blog, Ill
just address some key words for each.
Remember:

Ulcerative colitis Crampy pain. Mucosa and submucosa. Pseudopolyps.


HLA-B27. Ankylosing spondylitis. Pyoderma gangrenosum. Primary
sclerosing cholangitis. Toxic megacolon. Adenocarcinoma.
Crohns disease Colicky pain. Transmural. Lymphocytes. Granulomas.
Rectal sparing. Skip lesions. Fistulas. Strictures. B12 deficiency.

Both conditions usually present with diarrhea on the boards, so be able to quickly
recognize these key words for some easy points. Dont forget, both of these conditions
have an increased risk of colon cancer estimated to be 1-2% per year after 8 years of
disease. Thus surveillance colonoscopy should be started in patients with IBD for 8
years or longer.
6. Colon
Colorectal cancer is the second leading cause of cancer death in the United States.
According to the CDC, every year about 140,000 Americans are diagnosed with
colorectal cancer and over 500,000 die from it. That being said, it should come as no
surprise that colon cancer is a major GI topic in the ABIM exam.
Remember these Autosomal Dominant conditions and their management:

Familial adenomatous polyposis (FAP) caused by a mutation in the


APC gene
22 | P a g e

K N O W M E D G E

annual flex sig beginning at age 10-12, and colectomy should be


considered when polyposis is detected
Hereditary nonpolyposis colorectal cancer (HNPCC), or Lynch syndrome
caused by a mutations in the MLH1 and MSH2 mismatch repair genes
colonoscopy every 1 to 2 years starting at age 20-25 years or 10
years before the age at diagnosis of the youngest family member
with colon cancer
Peutz-Jeghers syndrome caused by a germ line mutation in the STK11
gene
Juvenile polyposis syndrome caused by a germ line mutation of the
SMAD4 gene

7. Liver
Last but definitely not least of the major parts of the digestive tract the liver. This
organ, weighing in at about 3 lbs, is the second largest organ (after the skin) and affects
nearly every physiologic process of the human body.
For the ABIM board exam, be able to interpret Hepatitis B serologies and have a good
understanding of the difference between acute and chronic infection.
Remember:

The goal of therapy in chronic Hepatitis B is suppression of viral


replication, seroconversion of HBeAg, and decrease in hepatic
inflammation (as evidenced by an improvement in liver enzymes)
Once a patient is found to have a chronic Hepatitis B infection,
surveillance should be undertaken to prevent the development of
cirrhosis and HCC by ultrasound and a-fetoprotein level every 6-12
months

In addition to viral hepatitis, several other causes of hepatitis need to be considered.


Be able to distinguish between hepatocellular injury and cholestatic injury.
Remember:

23 | P a g e

K N O W M E D G E

Hepatocellular elevation in ALT and AST released from injured


hepatocytes
Viral hepatitis, alcoholic hepatitis, drug-induced hepatitis, NASH,
ischemic hepatitis, autoimmune hepatitis (look for ASMA!),
hemochromatosis, Wilson disease, a1-antitrypsin deficiency
Cholestatic elevation in alkaline phosphatase occurs due to a decrease
in the flow of bile
Primary biliary cirrhosis (look for antimitochondrial antibodies!),
primary sclerosing cholangitis, drug-induced cholestasis

These are just a few key points to help you digest the Gastroenterology and Hepatology
section of the ABIM Internal Medicine exam. Good luck!

24 | P a g e

K N O W M E D G E

General Internal Medicine Pearls: Eye Conditions


By: Dr. Sunir Kumar
While most of the ABIM Examination topics fall neatly into organ system categories, not all of
them fit into this schematic. These include: ophthalmology, primary care screening
guidelines, vaccinations, etc. Here at Knowmedge, weve incorporated this important group
of subject areas into General Internal Medicine, similar to the American College of
Physicians Internal Medicine In-Training Exam Blueprint.
In this first of a series of blog posts, we review the key eye diseases: Conjunctivitis,
Macular Degeneration, Glaucoma, Retinal Detachment, and Cataracts. The eyes may
see only what the mind knows, but your mind should know these eye conditions for the
ABIM exam.
1. Conjunctivitis
Conjunctivitis is broken down into viral, bacterial, and allergic conjunctivitis.

25 | P a g e

K N O W M E D G E

Viral

Usually caused by adenovirus

Having a preceding upper respiratory infection or recent exposure to a person with


conjunctivitis are clues to aid in the diagnosis
Acute onset
Usually unilateral redness
Watery discharge is present
Highly contagious
Frequent hand washing must be performed to prevent spread of infection

Supportive treatment including cold compresses and artificial tears. NO role for
antibiotic eye drops with viral conjunctivitis

Bacterial

Common causing agents are Staph aureus, Streptococcus pneumoniae, or

Haemophilus influenzae. People who wear contact lenses can be infected by


Pseudomonas aeruginosa
Presence of mucopurulent discharge and crusting can occur in the morning when the

discharge is dry
Usually leads to redness of unilateral eye but may have bilateral involvement if
spread occurs by rubbing both eyes
Should be treated with antibiotics (e.g. erythromycin ointment and/or polymyxintrimethoprim drops) for 5-7 days
Patients who wear contact lenses should not wear contact lenses for at least 7 days
If patient has bacterial conjunctivitis for 4 weeks, this is chronic in nature and should
be evaluated by an ophthalmologist

Allergic

Can mimic viral conjunctivitis in terms of having clear (or ropy) discharge but

predominant symptom is eye itching that corresponds to specific seasons


Treatment is with oral anti-histamines, topical anti-histamines, and artificial tears

26 | P a g e

K N O W M E D G E

2. Macular Degeneration

Common in elderly individuals


Can lead to visual loss
Progression of disease can lead to difficulty reading, driving, or performing activities
of daily living
Two sub-types of macular degeneration are dry (atrophic) and wet (neo-vascular).
Most cases are dry macular degeneration. If, however, patient has wet macular
degeneration, it is more serious and can lead to blindness in an acute fashion rather
than progressive fashion.
Biggest risk factors are age, family history, cardiovascular disease, and smoking
Quitting smoking reduces the risk of developing macular degeneration and some
studies suggest that diet high in anti-oxidants can be protective. Patients can also
benefit by using a magnifying glass.

3. Glaucoma
Glaucoma is a condition resulting from increased intra-ocular pressure due to blockage in
drainage of aqueous humor. It can be broken down into primary open angle glaucoma
(POAG) and acute angle closure glaucoma.

POAG

Most common form of glaucoma and most common cause of irreversible blindness in
the world
Painless loss of peripheral vision that is gradual in nature. Later on, it can affect
central vision
Optic cup: disc ratio >0.5
Risk factors include age greater than 40, African-American race, and positive family
history
Mainstay of treatment is with pharmacological eye agents such as beta blockers,
carbonic anhydrase inhibitors, adrenergic agonists, hyperosmotic agents, and
prostaglandin analogues

27 | P a g e

K N O W M E D G E

Acute Angle Closure Glaucoma

More severe than POAG

Red eye, severe pain, headache, nausea and vomiting and visual halos
Intra-ocular pressure can be as high as 50 mmHg
When suspected, immediate referral to an ophthalmologist is required to prevent
permanent optic nerve atrophy

4. Retinal Detachment

Patients may present with floaters, squiggly lines, or flashes of light

This is an ophthalmology emergency and requires immediate attention by an


ophthalmologist

5. Cataracts

Opacity of lens will occur


Symptoms include painless decreased visual acuity, decreased night vision, glare,
and sometimes double vision
Physical examination can reveal a decreased or absent red reflex
Treatment is surgical removal of the cataract

Once again, the folks who write the Internal Medicine licensing exams dont expect you to
have the depth of knowledge regarding the plethora of eye diagnoses that an ophthalmologist
possesses. However, topics such as the ones mentioned in the pearls above should assist
you with the General Internal Medicine section of the med school clerkship shelf, ACP intraining and ABIM board exams.

28 | P a g e

K N O W M E D G E

General Internal Medicine Pearls: Vaccinations


By: Dr. Sunir Kumar
While most of the ABIM Examination topics fall neatly into organ system categories, not all of
them fit into this schematic. These include: primary care screening guidelines, substance
abuse, vaccinations, etc. Here at Knowmedge, weve incorporated this important group of
subject areas into General Internal Medicine, similar to the American College of Physicians
Internal Medicine In-Training Exam Blueprint. Earlier this week, we reviewed the 5 key eye
diseases seen on the ABIM board and NBME shelf exams. Today we explore the high-yield
topic of vaccinations.

A comprehensive adult immunization schedule, by Vaccine and Age Group, can be found on
the Centers for Disease Control and Prevention website (unlike other preventive health
measures, vaccination guidelines are not released by the USPSTF). Fortunately, you dont
need to memorize the entire CDC chart. Instead, for the ACP Internal Medicine in-training
exam, ABIM boards and NBME shelf exam, focus on these 7 vaccinations.

29 | P a g e

K N O W M E D G E

1. Tetanus, diphtheria, pertussis (Tdap)

Administer Tdap to all adults who have not previously received Tdap or whose

vaccination status is unknown.


Boost with Td every 10 yrs.

2. Influenza

All individuals age 6 months and older should be vaccinated yearly since the targeted
influenza strains changes on a yearly basis.

3. Herpes zoster (Shingles)


Adults 60 years of age and older should receive a single dose of the zoster vaccine.
This is regardless of whether the patient has had a prior episode of herpes zoster.
2. The vaccine is approved by the Food and Drug Administration (FDA) for patients 50
years of age and older, at this point, the CDC recommends the vaccination beginning
at age 60 years so stick with this guideline.
3. Patients with severe immunodeficiency should not receive the zoster vaccine.
1.

30 | P a g e

K N O W M E D G E

4. Pneumococcus

All adults 65 year of age and older should be recommended to have one-time
vaccination against Pneumococcus.
Pneumococcal vaccine should also be given to younger individuals who are current
smokers, nursing home residents, or have asthma/COPD, diabetes mellitus, chronic
renal failure, chronic liver disease, chronic cardiovascular disease, cochlear implants,
anatomic asplenia, or alcoholism.

5. Hepatitis A
Candidates for immunization against hepatitis A virus include:

Travelers to endemic areas

Illicit drug users


Men who have sex with men
Patients with chronic liver disease
Any patient seeking protection from HAV

6. Meningococcus
Only certain populations should be vaccinated against meningococcus:

College students living in dormitories


Patients with HIV
Asplenia

7. Human Papillomavirus (HPV)


Both males and females ages 11-26 are recommended to undergo the full series of the HPV
vaccination. The vaccine is not recommended for use in pregnant women.
Source: http://annals.org/article.aspx?articleid=1567229

31 | P a g e

K N O W M E D G E

5 Hematology Pearls
By Dr. Salim Rezaie
Hematology is a subject that comes up daily in clinical practice, and is a favorite on the ABIM
Internal Medicine board exam. Evaluation of anemia, as well as some other hematologic
disorders, is paramount for an internist to master. I will try and provide five evidence-based
pearls in this post that will help physicians understand some important concepts and avoid
common pitfalls in the recognition and treatment of such hematological disorders.
Pearl #1: Patients on iron supplementation SHOULD NOT have positive guaiac tests.

Studies in vitro show ferric iron (Fe3+) will give a positive guaiac reaction and ferrous
iron (Fe2+) does not

Iron is digested in the ferrous form and carried in the blood in the ferric form

Patients on iron supplementation with positive guaiac require screening for identifying
the source of gastrointestinal hemorrhage

Ferrous (Fe2+) iron does not cause positive guaiac tests in vivo

Pearl #2: The pentad of Thrombotic Thrombocytopenic Purpura (TTP) is not always
present.

The pentad is: microangiopathic hemolytic anemia (MAHA), thrombocytopenia, renal


abnormalities, neurologic abnormalities, and fever

Less than 50% of patients have the complete pentad

Measurement of ADAMTS13 activity is not required to make the diagnosis; the


diagnosis is clinical

The gold standard treatment is plasma exchange and if not available you may use fresh
frozen plasma as an alternative treatment

Pearl #3: Primary hemostasis disorders are a platelet dysfunction and secondary
hemostasis disorders are a clotting factor disorder.
32 | P a g e

K N O W M E D G E

Primary hemostasis Disorders:


o A result of platelet function
o Immediate clotting
o Patients will have petechiae and purpura
o All will have elevated bleeding time (platelets dont work) and normal PT/PTT
(no problem with clotting factors)

Secondary hemostasis Disorders:


o A result of clotting factors
o Delayed clotting (help strengthen clots by fibrin formation)
o Patients will have hematomas and hemarthroses
o All will have normal bleeding time (platelets work fine) and abnormal PT
(extrinsic pathway) and PTT (intrinsic pathway)

Pearl #4: Acute myelogenous leukemia (AML) type M3 has a good prognosis.

AML is the most common type of acute leukemia in adults

Typically M2 M5 types are myeloperoxidase stain positive (Remember that PTU and
micropolyangitis can also be positive)

Auer rods are pathognomonic for AML

Type M3 (promyelocytic) leukemia has t(15,17)

The treatment of choice is all-trans retinoic acid (ATRA)

The single most important prognostic factor in AML is cytogentetics: t(15;17) has a 70%
5 year survival and 33% relapse rate

Pearl #5: Anemia is the most common hematologic abnormality, so know it cold.

Iron deficiency anemia is the worlds most common cause of anemia

Iron deficiency anemia:


o Low iron, transferrin saturation, and ferritin
o Elevated TIBC

33 | P a g e

K N O W M E D G E

o Transferrin Receptor Index = transferrin receptor/Log Ferritin is the most


sensitive assay for iron deficiency anemia (>2.0 = Iron Deficiency Anemia; <1.0
= Anemia of Chronic Disease)
o Treatment is PO iron if no improvement after 6 weeks consider IV iron
o The earliest lab to check after starting iron replacement is the reticulocyte count
(Begins to increase at about 5 to 7 days)
o Ascorbic acid (vitamin C) supplementation increases absorption of iron
o Most iron is absorbed in the duodenum
o Celiac sprue can cause iron deficiency anemia: Best test is tissue
transglutaminase antibody or antiendomysial antibody; Tx is a gluten free diet;
Can see dermatitis herpetiformis associated with this entity
As I have stated in my high yield pearls to pass the boards: Most commons are emphasized
on the ABIM curriculum, so it is good to know these disease processes. Also, diseases that
are increasing in incidence or have a high mortality associated with them are also high yield.
Anemia (most common hematologic abnormality), AML (most common acute leukemia in
adults), and Thrombotic Thrombocytopenic Purpura (high mortality) are 3 of the hematology
pearls covered in this post. Best of luck as you are preparing for your ABIM board exam, and
hopefully these pearls help you get a few more questions correct.
References:
1. George JN. How I Treat Patients With Thrombotic Thrombocytopenic Purpura: 2010.
Blood 2010 Nov; 116 (20): 4060 9. PMID: 20686117
2. Grimwalde D et al. The Importance of Diagnostic Cytogenetics on Outcome in AML:
Analysis of 1,612 patients entered into the MRC AML 10 Trial. The Medical Research
Council Adult and Childrens Leukemia Working Parties. Blood 1998 Oct; 92 (7): 2322
33. PMID: 9746770
3. Konrad G et al. Are Medication Restrictions Before FOBT Necessary?: Practical Advice
Based on a Systematic Review of the Literature. Can Fam Physician 2012 Sep; 58 (9):
939 48. PMID: 22972722
4. McDonnell WM et al. Effect of Iron on the Guaiac Reaction. Gastroenterology. 1989
Jan; 96 (1): 74 8. PMID: 2909440
34 | P a g e

K N O W M E D G E

5. Rimon E et al. Diagnosis of Iron Deficiency Anemia in the Elderly by Transferrin


Receptor-Ferritin Index. Arch Intern Med 2002 Feb; 162 (4): 445 -9. PMID: 11863478

35 | P a g e

K N O W M E D G E

6 Infectious Disease Pearls


By Dr. Salim Rezaie
In keeping with the popularity of the high-yield pearls posts from nephrology, I decided to write
a post giving you my 5 high-yield infectious disease (ID) pearls, beneficial for your clinical
practice and ABIM exam preparation. The purpose of these posts is to give you some concise,
practical teaching points that are supported by the medical literature.
Pearl # 1: Staphylococcus aureus bacteremia can be more complicated than getting 2
weeks of intravenous antibiotics

Patients treated for at least 2 weeks of antibiotics are more likely to be cured of
bacteremia and less likely to have recurrence of bacteremia3
Recurrence of bacteremia may be due to bone, joint, and cardiac involvement due to
inadequate initial antimicrobial course3
Patients with cardiac, bone, and joint involvement need at least 4 6 weeks of
antimicrobial coverage
Remove intravascular catheters if patient has Staphylococcus aureus bacteremia (20
26% of cases are complicated by infective endocarditis or metastatic infection) 8
Patients with hematuria during Staphylococcus aureus bacteremia should have further
evaluation for infective endocarditis8
Oral linezolid can be used to complete a 2-week course in some cases12

Pearl # 2: Candida in the blood is not a contaminant, but could be in the sputum or
urine

Candida in the blood is NEVER a contaminant 11


Removal of an intravenous catheter alone is never an absolute treatment; highest
mortality rates are seen in patients without antifungal therapy9

All patients with candidemia should undergo ophthalmologic examination 10


Micafungin is the treatment of choice for candidemia, and preferred over azoles8
Asymptomatic candiduria does not require further workups or antifungal therapy in most
cases; Symptomatic funguria always requires treatment 6

36 | P a g e

K N O W M E D G E

Pearl # 3: Clostridium difficile toxin should not be re-checked for cure and has a poor
sensitivity

Clostridium difficile associated diarrhea (CDAD) should be suspected after recent


antibiotic use and/or if in the hospital for more than 2 days
Consider infection with C. difficile in the differential diagnosis when a leukocytosis in
hospitalized patients develops
Diarrhea does not have to be present to have a diagnosis of C. difficile colitis
C. Diff stool assay produces a false negative test 10 20% of the time2
C. Diff stool toxin assays remain positive during and after successful treatment,
therefore follow up assays for cure are not helpful, follow clinical course 7

Pearl #4: Blood cultures should always be obtained before parenteral antibiotics are
given

Both the Infectious Disease Society of America (IDSA) and American Thoracic Society

(ATS) advocate obtaining two sets of blood cultures prior to initiating antibiotic therapy
Coagulase-negative staphylococci is a contaminant in blood cultures about 82% of the
time11

The difference between blood cultures before the initiation of antibiotics and after the
initiation of antibiotics in identifying a pathogen is 40% versus 18.7%4
Appropriate blood cultures, allows for prompt identification of the offending organisms
which influences diagnosis, therapy, and prognosis when positive

Pearl #5: Empiric antibiotics for acute uncomplicated cystitis have changed5

Nitrofurantoin monohydrate/macrocrystals 100mg BID for 5 days is the appropriate


choice for empiric therapy of urinary tract infection
Trimethoprim-sulfamethoxazole 160/800mg BID for 3 days is an appropriate empiric
choice if local resistance rates of uropathogens do not exceed 20% (expert opinion)
Fosfomycin trometamol 3g in a single dose is an appropriate empiric choice for urinary
tract infection, but may be inferior efficacy compared to standard short-course regimens
Pivmecillinam 400mg BID for 3 7 days is also an appropriate empiric antimicrobial
agent where available

37 | P a g e

K N O W M E D G E

Fluoroquinolones (ofloxacin, ciprofloxacin, and levofloxacin) should be considered


alternative antimicrobials for acute uncomplicated cystitis
Amoxicillin or ampicillin should not be used for empirical treatment due to resistance to
these agents

Pearl #6: The loading dose of vancomycin is 25 30mg/kg based on actual body weight
in critically ill patients

Best predictor of efficacy of vancomycin is time above the antimicrobial MIC1

There are so many more high-yield pearls for infectious disease, but these in my humble
opinion are practice changing, cost saving, and also affect patient outcomes.
References:
1. Ackerman BH et al. Necessity of a Loading Dose When Using Vancomycin in Critically
Ill Patients. Journal of Antimicrobial Chemotherapy. 1992; 29 (4): 460 1. PMID:
1607335
2. Bartlett JG et al. Antibiotic-Associated Diarrhea. NEJM 2002 346: 334 339. PMID:
11821511
3. Fowler VG et al. Outcome of Staphylococcus Aureus Bacteremia According to
Compliance With Recommendations of Infectious Diseases Specialists: Experience
With 244 Patients. Clin Infect Dis 1998 Sep; 27 (3): 478 86. PMID: 9770144
4. Grace CJ et al. Usefulness of Blood Culture for Hospitalized Patients who are
Receiving Antibiotic Therapy. Clin Infect Dis 2001 Jun; 32 (11): 1651 5. PMID:
11340541
5. Gupta et al. International Clinical Practice Guidelines for the Treatment of Acute
Uncomplicated Cystitis and Pyelonephritis in Women: A 2010 Update by the Infectious
Diseases Society of America and the European Society for Microbiology and Infectious
Diseases. Clin Infect Dis 2011 Mar; 52 (5): e103 20. PMID: 21292654
6. Hollenbach E. To Treat or Not to Treat Critically Ill Patients with Candiduria. Mycoses
2008 Sep. 51; (2) 12 24. PMID: 18721329
7. Kelly CP et al. Clostridium difficile More Difficult than Ever. NEJM 2008 Oct; 359 (18):
1932 -40. PMID: 18971494

38 | P a g e

K N O W M E D G E

8. Kim AI et al. Staphylococcus Aureus Bacteremia: Using Echocardiography to Guide


Length of Therapy. Cleve Clin J Med 2003 Jun; 70 (6): 517, 520 -1, 525 -6. PMID:
12828223
9. Nguyen MH et al. Therapeutic Approaches in Patients with Candidemia. Evaluation in
a Multicenter, Prospective, Observational Study. Arch Intern Med 1995; 155 (22): 24
29. PMID: 7503601
10. Pappas PG et al. Clinical Practice Guidelines for the Management of Candidiasis: 2009
Update by the Infectious Diseases Society of America. Clin Infect Dis 2009 Mar; 48 (5)
503 35. PMID: 19191635.
11. Pien BC et al. The Clinical and Prognostic Importance of Positive Blood Cultures in
Adults. Am J Med 2010 Sep; 123 (9): 819 28. PMID: 20800151
12. Sharpe JN et al. Clinical And Economic Outcomes of Oral Linezolid Versus Intravenous
Vancomycin in the Treatment of MRSA-complicated, Lower-Extremity Skin and SoftTissue Infections caused by Methicillin-Resistant Staphylococcus Aureus. Am J Surg
2005 Apr; 189 (4): 425 8. PMID: 15820454

39 | P a g e

K N O W M E D G E

11 Nephrology Pearls
By Dr. Salim Rezaie
Recently, I read an article on some very useful chronic kidney disease (CKD) pearls to help
those healthcare providers who are not nephrologists care for their patients and also prepare
for the ABIM Internal Medicine Board exam at the same time. The article was titled The Top
10 Things Nephrologists Wish Every Primary Care Physician Knew by Paige NM et al and
basically stated: early recognition of kidney disease is essential in order to begin
measures to prevent progression and complications such as kidney failure,
cardiovascular disease, and premature death. I have decided to break the content into two
parts; the first half will be discussed in this post:
Pearl # 1: A Normal Creatinine Level May Not Be Normal

Make sure to take muscle mass, age, sex, height, and limb amputation into account

Consider using MDRD or Cockcroft-Gault equations to calculate glomerular filtration


rate (GFR)
MDRD and Cockcroft-Gault equations are imprecise at high values for GFR (low values
for serum creatinine)

Pearl # 2: Know the Medications That Falsely Elevate Serum Creatinine Levels

Trimethoprim-sulfamethoxazole and cimetidine decrease secretion of creatinine

Both medications can increase creatinine level by as much as 0.4 0.5mg/dL


An increase in creatinine level is a true decrease in GFR only if there is also a
corresponding increase in BUN

Pearl # 3: Patients with Decreased GFR or Proteinuria Need to be Evaluated for the
Cause

Urine dipstick detects concentration of albumin in urine


Urine concentration can affect dipstick results therefore a quantitative estimation of
proteinuria is required to evaluate dipstick proteinuria
40 | P a g e

K N O W M E D G E

The preferred quantitative test is spot urine protein to creatinine ratio (accurate & more
convenient than 24-hr urine collection)
A urine protein to creatinine ratio 1 has a higher risk of progression of CKD

Pearl # 4: Early-Stage CKD Should Have Periodic Evaluation and Intervention to Slow
Progression

Try and avoid nephrotoxic agents (NSAIDs, aminoglycoside antibiotics, and


radiocontrast)

Monitor and control blood pressure with a goal of <130/80 mmHg


Angiotensin-converting enzyme inhibitors (ACEIs) and angiotensin II receptor blockers
(ARBs) may slow progression of CKD, especially in patients with proteinuria
Monitor phosphorous, calcium, and parathyroid hormone levels in all patients with
stage 3 to 4 CKD
Patients with CKD are at higher risk of cardiovascular events and should be on a baby
aspirin, and a lipid lowering agent with goal LDL <100mg/dL (Maybe <70mg/dL for LDL
in patients with CAD and CKD)
Consider referral and co-management with a nephrologist if a patient has CKD
progression, active urine sediment and/or stage 3 CKD
ALL patients with Stage 4 5 CKD should be referred to a nephrologist

Pearl # 5: DO NOT Discontinue an ACEI or ARB Because of a Small Increase in Serum


Creatinine or Potassium

Both ACEIs and ARBs are the drugs of choice to prevent progression of proteinuric

CKD
An increase of 20 to 30% of the creatinine level is acceptable
Just make sure to confirm the creatinine stabilizes and does not continue to increase

Also a serum potassium of 5.5 mEq/L is acceptable as long as it is stable and as long
as the patient is aware of dietary restrictions
Serum creatinine and potassium levels should be ordered within one week of increase
in dose of ACEI or ARB
If a patient has an increase in creatinine from 1.5 to 1.9 (<30% increase) CONTINUE
THE ACEI
41 | P a g e

K N O W M E D G E

If the same patient has an increase in creatinine from 1.5 to 2.2 (>30% increase) STOP
THE ACEI

Pearl # 6: Anemia in Patients with CKD Should be Treated, but not Overtreated

Anemia of Chronic Disease can lead to fatigue, left ventricular hypertrophy, and
increased risk of cardiovascular events
Hemoglobin target for CKD should be between 11 12 g/dL NOT to exceed 13g/dL
Overcorrection of hemoglobin can result in higher risk of stroke, thrombosis, and
hypertension
Correct all other reversible causes of anemia

Pearl # 7: Phosphate-Containing Bowel Preps Should be Used With Caution

Sodium phosphate bowel preparations are more convenient than some other preps
(Easier to use)
However, some studies have suggested that they can cause phosphate nephropathy
leading to AKI or worsening CKD
Instead use polyethylene glycol for the bowel prep (only downside is the volume that
has to be consumed; Does not cause volume or electrolyte shifts)

Pearl # 8: Patients With Severe CKD Should Avoid Magnesium- or AluminumContaining Preparations

These include over-the-counter agents such as Maalox and Mylanta

Use of these agents can lead to hypermagnesemia, acute aluminum toxicity, worsening
renal function, bone disease, and neurotoxicity
The preferred quantitative test is spot urine protein to creatinine ratio (accurate & more
convenient than 24-hr urine collection)
A urine protein to creatinine ratio 1 has a higher risk of progression of CKD

Pearl # 9: Most Patients With Hypertension Should NOT Be Screened for Secondary
Hypertension, But be Aware of Certain Clinical Clues

42 | P a g e

K N O W M E D G E

In general 95% of patients have primary or essential hypertension, and only 5% have

a secondary cause
Clues include: Severe or difficult to control HTN, HTN that suddenly develops, or HTN
that is associated with other clinical findings are some clues
Hypokalemia: Consider primary hyperaldosteronism
Headaches, palpitations, and sweats: Consider Pheochromocytoma
Moon facies and/or striae: Consider Cushing Syndrome
History of snoring in obese patient: Consider Obstructive Sleep Apnea
Bruit on one side of the abdomen: Consider Renal Artery Stenosis

Over-the-counter medications (NSAIDs, Birth Control Pills, or Decongestants)


Non-compliance with Diet (High Sodium Intake)

Pearl # 10: Recurrent Nephrolithiasis, Needs a Metabolic Evaluation to Identify and


Treat Modifiable Risk Factors

Nephrolithiasis recurrence over a 10 year period for calcium oxalate stones is about
50% without treatment
Family History of nephrolithiasis, inflammatory bowel disease, frequent urinary tract
infections, or history of nephrocalcinosis should be referred to a nephrologist
Initial workup should start with: diet history, medications, serum calcium, phosphorous,
electrolytes and uric acid

Pearl # 11: Cyclosporine and Tacrolimus (Calcineurin Inhibitors) Have Many Drug-Drug
Interactions

Any new medication or supplement that a post-kidney transplant patient requests

should be reviewed first before prescribing


St. Johns Wort, rifampin, phenytoin, and carbamazepine can all lower cyclosporine
levels

Diltiazem, verapamil, and erythromycin can increase cyclosporine levels


Cyclosporine can interfere with certain statins such as simvastatin, increasing the risk
of statin-induced rhabdomyolysis

43 | P a g e

K N O W M E D G E

References:
1. Bakris GL et al. Angiotensin-Converting Enzyme Inhibitor-Associated Elevations in
Serum Creatinine: Is this a Cause for Concern? Arch Intern Med. 2000; 160 (5): 685
693. PMID: 10724055
2. Douglas K et al. Meta-analysis: The Effect of Statins on Albuminuria. Ann Intern Med.
2006; 145 (2): 117 124. PMID: 16847294
3. Levey AS et al. Definition and Classification of Chronic Kidney Disease: A Position
Statement from Kidney Disease: Improving Global Outcomes (KDIGO). Kidney Int.
2005; 67 (6): 2089 2100. PMID: 15882252
4. Paige NM et al. The Top 10 Things Nephrologists Wish Every Primary Care Physician
Knew. May Clin Proc. 2009 Feb; 84 (2): 180 186. PMID: 19181652

44 | P a g e

K N O W M E D G E

7 Oncology Pearls
By: Dr. Sunir Kumar
Oncology is an essential part of the Internal Medicine Medical Clerkship and ABIM Board
exam. According to the ABIM exam blueprint, questions testing Oncology topics comprise
~7% of the exam. Initially, oncology seems like a very difficult subject given how the treatment
regimens of the many types of carcinomas are ever-changing. However, realize that Internal
Medicine exams are focused on the basics of the different malignancies. These 8 pearls will
get you off to a good start for identifying
Pearl # 1: Which cancers correspond to lytic bone lesions? Blastic? Lytic or Blastic?

Lytic bone lesions


o Melanoma
o Multiple Myeloma
o Renal cell cancer
o Thyroid cancer
Blastic bone lesions
o Prostate cancer
Lytic or blastic lesion
o Lung cancer
o Breast cancer

45 | P a g e

K N O W M E D G E

Pearl # 2: Which paraneoplastic syndrome goes with which cancer?

Pearl # 3: Know these high-yield Breast Cancer facts for the ABIM and IM shelf exams

If the lesion is >1cm OR the patient has lymph node involvement chemotherapy will
be part of the treatment protocol.
If sentinel node biopsy is negative no further lymph node biopsy is required
If the patients breast cancer is ER/PR (+) and patient is pre-menopausal Tamoxifen
is given for 5 years
If the patients breast cancer is ER/PR (+) and is post-menopausal Aromatase
inhibitors (eg. Anastrozole) is given
If the patients breast cancer is HER2/neu (+) Trastuzumab is given can cause
CHF check echocardiogram to assess left ventricular function
46 | P a g e

K N O W M E D G E

Triple negative breast cancer (ER negative, PR negative, HER2/neu negative) has the
worst prognosis

Pearl # 4: Dont let chromosomal translocations overwhelm you

t (14,18) Follicular Lymphoma

t (8,14) Burkitts Lymphoma


t (9,22) Known as Philadelphia Chromosome (BCR-ABL) gene fusion good

prognosis in CML patients and bad prognosis in ALL patients


t (15,17) Acute promyelogenous Leukemia (M3) good prognosis Treatment is
with All-Trans Retinoic Acid (ATRA)
t (11,14) Mantle cell Lymphoma highly aggressive form of Non-Hodgkins
Lymphoma cyclin D1 oncogene is overexpressed treat with R-CHOP

47 | P a g e

K N O W M E D G E

Pearl # 5: Testicular cancer is broken down into seminomas and non-seminomas

Seminomas are Radiation sensitive and usually have a normal AFP and HCG level

(although HCG can be elevated in 5-10% of cases)


Non-Seminomas (e.g., yolk sac tumor, embryonal carcinoma, teratoma,
choriocarcinoma) are radiation resistant and usually have elevated AFP and HCG level

Pearl # 6: High-yield facts about Colon Cancer for the ABIM and IM Shelf Exam

If cancer invades mucosa or sub-mucosa colon resection is required

If cancer invades muscularis propria, serosa, or has LN involvement colon resection


is required +chemotherapy (usually FOLFOX) +/- Irinotecan. FOLFOX is 5 Flurouracil,
Oxaliplatin, and Leucovorin
Most common area of metastasis for colon cancer is the liver

If solitary liver lesion resection of liver can be performed


If multiple liver lesions poor prognosis and survival rate is low

Pearl # 7: Must know Oncological Emergencies

48 | P a g e

K N O W M E D G E

Superior Vena Cava Syndrome caused by obstruction to the superior vena cava
leading to:
o Neck and facial swelling
o Dyspnea
o Cough
o Physical examination will show distended jugular veins
o Chest x-ray may reveal mediastinal widening or right hilar mas
o Most common cause is lung cancer (particularly small cell lung cancer).
Malignant lymphoma is also a well-known cause of superior vena cava
syndrome
o Management obviously requires treatment of the underlying malignancy but
symptomatic measures such as diuretics and elevation of the bed usually suffice

Spinal cord compression


o Most common cancers that cause spinal cord compression are: lung cancer,
breast cancer, prostate cancer, multiple myeloma, and lymphoma
o Back pain is the most commonly presenting symptom. Other concerning
symptoms include muscle weakness, numbness/tingling, cauda equina
syndrome, and loss of bowel or bladder control
o Diagnosis is required promptly to prevent further neurological deterioration and
an MRI of the entire spine should be ordered
o Management for spinal cord compression is with Dexamethasone 10 mg IV X 1
dose, followed by Dexamethasone 4 mg IV every 6 hours
o Radiation therapy can help in shrinking the tumor; however, recent studies
indicate that surgical decompression is superior to RT. Therefore, an immediate
neurosurgical consultation is required when this condition is suspected or
diagnosed
Tumor Lysis Syndrome
o Intracellular contents leak out into the bloodstream
o Although this condition can occur with any malignancy, it is more common in
aggressive lymphomas and hematological malignancies
o Hyperkalemia, Hyperuricemia, Hyperphosphatemia, Acute renal failure, and
Hypocalcemia are often seen in patients with tumor lysis syndrome

49 | P a g e

K N O W M E D G E

o Management of tumor lysis syndrome requires electrolyte abnormality


correction, aggressive hydration, and allopurinol
Once again, the folks who write the Internal Medicine licensing exams dont expect you to have
the depth of knowledge regarding the plethora of cancer diagnoses that an oncologist possesses.
However, topics such as the ones mentioned in the pearls above should assist you with the
oncology section of the med school clerkship shelf and ABIM board exams.

50 | P a g e

K N O W M E D G E

5 Neurology Pearls
By: Dr. Sunir Kumar
Neurology is an extremely important part of the Internal Medicine Medical Clerkship and ABIM
Board exam. According to the ABIM exam blueprint, Neurology comprises ~4% of the exam.
Pearl # 1: In Neurology questions on the medical school clerkship and ABIM board
exam, nothing helps narrow the diagnosis than the reflexes mentioned in the vignette
Review the slide below for the most high-yield conditions associated with hyperreflexia, normal
reflexes, hyporeflexia, and delayed reflexes and absent reflexes.

51 | P a g e

K N O W M E D G E

Pearl # 2: Were not trained ophthalmologists but remembering these eye conditions
can add points to your ABIM score

Optic nerve lesion can lead to complete blindness in the ipsilateral eye (monocular
blindness of the ipsilateral eye)
Optic chiasm lesion Bitemporal hemianopiacommon in pituitary tumors that
compress the optic chiasm
Optic tract lesion contralateral homonymous hemianopia
Optic radiation lesion contralateral homonymous quadrantanopia

Lack of an ipsilateral adduction to a contralateral gaze is a Medial Longitudinal Fasciculus


(MLF) lesion. This condition, which is also known as intranuclear ophthalmoplegia, is seen in
patients who have Multiple Sclerosis.
In normal individuals, if asked to look the right, the right eye should abduct and the left eye
should adduct. If a patient with MS is asked to look to the right (for example), he/she will be
able to abduct the right eye but fails to adduct the left eye Lesion is Left MLF.
Same concept applies when asked to look to the left. Normally, the left eye will abduct and
the right eye should adduct. In patients with MS, patients lose the ability to adduct the right
eye Lesion is Right MLF.
Argyll Robertson Pupil eyes will be able to constrict when the patient focuses on a near
object (eg. bringing fingers to the nose). This is known as accommodation. However, patients
with an Argyll Robertson pupil lose the ability to constrict the eyes when bright light is shined
into their eyes. In a nutshell, the eyes cant react to light but can accommodate. This condition
is often seen in patients with syphilis.
Marcus Gunn Pupil This condition is also known as Relative Afferent Pupillary Defect
(RAPD). In normal individuals, when a swinging flashlight test is performed, both the direct
and consensual eye should constrict to light.

52 | P a g e

K N O W M E D G E

With Marcus Gunn pupil, lets suppose the left eye is affected. If light is shined into the right
eye, both the direct and consensual will constrict. When light is shined into the left eye, both
the direct and consensual eye will seem dilated (lack of constriction) Shows damage to the
ipsilateral optic nerve.
Pearl # 3: Know the indications and contraindications of use of t-PA.
Indications:
o Ischemic stroke as seen on CT head with CLEARLY defined onset of symptoms
o Time of onset of symptoms to administration of t-PA should be no later than 3
hours (180 minutes)

Major Contraindications:
o Blood pressure greater than or equal to 185/110mmHg
o CT head indicates a hemorrhagic stroke rather than an ischemic stroke
o Major trauma to the head within the past 3 months
o Major surgery within the past 14 days
o Current use of anticoagulants as administering t-PA with anticoagulants
increases risk of major bleeds
o Platelet count of less than 100,000/uL
o PT>15 seconds
o Glucose 400 mg/dl

53 | P a g e

K N O W M E D G E

Pearl # 4: Identifying buzzwords is key for selecting the correct neurological diagnosis
when CT/MRI findings are included in the vignette.

Multiple Sclerosis increased T2 signal and decreased T1 signal. There will be

increased enhancement of active lesions with gadolinium.


Multi-infarct dementia multiple hypo-dense areas without enhancement.
Toxoplasmosis, brain abscess, and lymphoma Ring enhancing lesions seen on CT

scan
Cerebral atrophy dilated ventricles with dilated sulci
Normal pressure hydrocephalus dilated ventricles without dilated sulci. Patient is
wet, wobbly, and weird. (urinary incontinence, ataxia, and dementia triad is often seen
in these patients)
Alzheimers DiseaseBrain atrophy with or without periventricular white matter lesions
54 | P a g e

K N O W M E D G E

Pearl # 5: Differentiating Myasthenia Gravis and Eaton-Lambert Syndrome can seem


challenging at first. Thats why theyre on the ABIM.
Ever find yourself second-guessing whether its Eaton-Lambert or Myasthenia Gravis that
improves with repetitive movements? And, which one is associated with thymoma? Before
letting your head spin or do cartwheels, take a few minutes to learn the difference between
these two neuromuscular disorders. The concise yet useful categorization will make it difficult
to get the two mixed up.
Myasthenia gravis

Antibodies to post-synaptic acetylcholine receptors

Ptosis and diplopia can be presenting symptoms


Can be associated with thymoma
Reflexes are normal
Power decreases with repetition

Eaton-Lambert Syndrome

Antibodies to pre-synaptic acetylcholine receptors

Ptosis and diplopia are usually absent


Usually associated with oat cell carcinoma (small cell carcinoma) of the lung
Reflexes are decreased (hyporeflexia)
Power improves with repetition (As you EAT, you get stronger)

Lets review it in Knowmedge slide form:

55 | P a g e

K N O W M E D G E

Once again, the folks who write the Internal Medicine med school clerkship shelf and ABIM
board exams dont expect you to have the depth of knowledge regarding neurological
conditions that a neurologist possesses. However, topics such as the ones mentioned in the
slides and pearls above should assist you with the neurology section of these exams.

56 | P a g e

K N O W M E D G E

Quick and Easy Guide to Selecting the Right Topical Steroid


By: Dr. Sheila Krishna
Several of the most frequently utilized dermatologic treatmentsespecially for conditions that
arise in primary care and thus may be found on the dermatology portion of the Internal
Medicine board and medical school shelf examsbelong to the topical corticosteroid
category.
As there are dozens of options of various strengths, preparation, and container sizes (along
with the consideration of whether patients prefer generic vs. brand, over-the-counter vs.
prescription), it can be extremely overwhelming to select the correct topical corticosteroid.
Fortunately, in most instances, there is more than one reasonable treatment option that will
provide the patient with the relief they need.
Lets begin by reviewing 5 Dermatologic pearls that will be helpful in your clinical practice and
may also help you select a couple more correct answers on the ABIM board exam.
Pearl # 1: Know the different strengths of the topical corticosteroids.
There are a total of 7 different classes of topical corticosteroids, with class 1 representing the
highest potency and class 7 representing the lowest potency. Its critical to be able to
recognize the medications potency because we need to find the best match for the patients
conditionstrong enough to treat the disease but gentle enough to avoid unnecessary side
effects. In Knowmedge fashion, heres a slide that gives the most commonly used topical
corticosteroids with their class number and relative potency.

57 | P a g e

K N O W M E D G E

Class 1 and 2 topical corticosteroids should never be prescribed on the face of body folds or
groin as these areas have more fragile skin that can be damaged by the skin-thinning potential
of these stronger topical agents.

Pearl # 2: When pressed for time, consider these selections.

For treating non-folded trunk and extremities, Triamcinolone 0.1% is a good option. For
treating the face and body folds (e.g. axilla, groin, inframammary areas, panniculus), Desonide
or Hydrocortisone can be used. For the palms and soles, Fluocinolone or Clobetasol may be
tried.

Pearl # 3: Select the correct preparation

58 | P a g e

K N O W M E D G E

Once you think you have the right medication selected, you may pause when asked by your
electronic prescription-ordering system or your local pharmacist which preparation you are
ordering: cream, ointment, or lotion. Make sure that the patient has been consulted about their
preferred preparation since they may not be adherent with the treatment plan if they find it
makes them feel uncomfortable.

From a cosmetic acceptance standpoint, creams tend to be an acceptable option since they
absorb well and lack the greasy texture of ointments. However, keep in mind that they may
sting if applied to open areas of the skin and they can contain preservatives that can lead to
an allergic contact dermatitis.

Ointments usually contain petrolatum, which provides more of an emollient effect to help retain
water. They are less likely to sting than creams. The biggest issue is the greasy texture that
some patients find uncomfortable. Lotions are more water-based than oil-based. As a result,
they are easily absorbed into the skin and spread easily, making them a good choice for
covering large areas of the body.

Pearl # 4: Finding the correct amount to prescribe

Unlike the straight-forward exercise of selecting the quantity of oral medications90 days of
a once daily medication equals 90 pillsproviding enough topical agent to last a patient until
the next scheduled refill is a bit more challenging. If too little is prescribed, they will be
frustrated by frequent trips to the pharmacy, not to mention the high co-pay they may be
dishing out each time they visit.

A good rule of thumb to use is that 1 palm is 1% of the body when estimating the % of the skin
that is affected by the patients dermatologic condition. Most topical corticosteroids come in
30 to 60 gram tubes. For a typical 70-kg patient, a 30gram tube provides one application to
the entire body.

59 | P a g e

K N O W M E D G E

Pearl #5: Dont forget the importance of gentle skin care

Atopic dermatitis and xerosis may not resolve despite topical corticosteroids if patients
continue to add agents to their skin which can increase drying. Harsh body washes and soap
and laundry detergents can do a number on the skin. Tell patients to avoid those and instead
apply emollients, which consist of water-supplying moisturizers (e.g. coconut oil, mineral oil)
and water loss-reducing occlusives (e.g. petroleum jelly).

60 | P a g e

K N O W M E D G E

8 Pulmonary Disease & Critical Care Pearls


By: Dr. Sunir Kumar
Pulmonary disease and critical care are an extremely important part of the Internal Medicine
Medical Clerkship and ABIM Board exam. According to the ABIM exam blueprint, questions
testing pulmonary disease topics comprises ~10% of the exam. That places it second only to
cardiologys 14% in terms of relative percentage.
Pearl # 1: ABGs do not need to stand for AnyBodys Guess
The next time you see an arterial blood gas (ABG) on a practice or actual exam question,
dont start searching for the Panic (or Skip) button. Determining diagnoses based on ABGs
can seem daunting at first but by following a simple yet systematic approach, we can tackle
these questions without difficulty. Click on the link to our previous How to Handle ABGs on
the ABIM Board Exam and after reviewing the 5 easy steps, it wont be long before you look
forward to ABG questions on the exam.
Pearl # 2: Obstructive or Restrictive? That is the question surrounding Lung Disease
To distinguish between obstructive and restrictive lung disease, the first value to look at on
the pulmonary function test (PFT) report is the Total Lung Capacity (TLC), which is defined as
the volume of air contained in the lungs at the end of a maximal inspiration.

Obstructive Lung Disease: TLC will be increased (example: asthma and COPD)

Restrictive Lung Disease: TLC will be decreased. Restrictive Lung Disease is further
sub-divided into intra-thoracic and extra-thoracic lung disease.
o

Intra-thoracic lung Disease (FEV1/FVC normal or increased above 80%, DLCO


decreased, Residual Volume decreased)
Sarcoidosis

Idiopathic Pulmonary Fibrosis


Hypersensitivity Pneumonitis
Pneumoconiosis
ABPA
Churg-Strauss Syndrome
Asbestosis
61 | P a g e

K N O W M E D G E

Silicosis
Berylliosis
Extra-thoracic Lung Disease (FEV1/FVC normal or increased above 80%,
DLCO normal, Residual Volume increased)
Obesity
Kyphosis
Myasthenia Gravis
Guillain-Barre Syndrome
Muscular dystrophy

Pearl # 3: The Mnemonic CHAD PARS helps recall the major causes of
bronchiectasis.

C CYSTIC FIBROSIS
H HYPOGAMMAGLOBULINEMIA
A ALPHA 1 ANTITRYPSIN DEFICIENCY
D DYSKINETIC CILIARY SYNDROME
P PNEUMONIA
A ALLERGIC BRONCHOPULMONARY ASPERGILLOSIS (ABPA)
R RHEUMATOID ARTHRITIS
S SJOGRENS SYNDROME

Pearl # 4: Not all that wheezes is asthmabut it sure is important for the ABIM boards
and medicine shelf exam
Frequency of asthma symptoms provides the necessary information for proper classification
of this reversible obstructive lung disease (defined by greater than or equal to 12% increase
in the FEV1 after use of an albuterol inhaler). Only then can we determine the best treatment
for the affected patient in an exam vignette.
This slide conveniently depicts the criteria for intermittent, mild persistent, moderate
persistent, and severe persistent asthma, along with their recommended treatments.

62 | P a g e

K N O W M E D G E

Pearl # 5: Similarly, mastering COPD classification (using the GOLD criteria) and
treatment is essential
Unlike asthma, COPD is an irreversible condition. Administering the bronchodilator albuterol
will not increase FEV1.
GOLD (Global Initiative for Chronic Obstructive Lung Disease) criteria is the primary method
used to diagnose and identify the severity of COPD. A diagnosis of COPD should be
considered for any patient over the age of 40 who has any of the following conditions:

Dyspnea that is persistent, worsens over time and gets worse with exercise

Chronic cough
Chronic sputum production

63 | P a g e

K N O W M E D G E

History of exposure to risk factors (Tobacco smoke, smoke from home cooking,
occupational dust, chemicals)
Family history of COPD

FEV1/FVC ratio less than 70% is an indication that there is an airflow limitation and, thus,
COPD. The spirometric criteria for a diagnosis of COPD is a post-bronchodilator FEV1/FVC
ratio less than 70%.
FEV1 will tell us the intensity of the COPD which can be characterized into four stages:

Stage I (Mild): FEV1 > 80% of predicted value; Rx: Short acting Bronchodilator as
needed with or without Ipratropium
Stage II (Moderate): 50% FEV1 < 80% of predicted value; Rx: Short acting
Bronchodilator as needed with long acting bronchodilator around the clock with or
without pulmonary rehab
Stage III (Severe): 30% FEV1 < 50% of predicted value; Rx: As above for moderate
COPD plus Inhaled steroids
Stage IV (Very severe): FEV1 < 30% of predicted value (or FEV1 < 50% of predicted
value plus chronic respiratory failure); Rx: As above for severe COPD plus Long-term
oxygen therapy for at least 15 hours daily. Surgical intervention should be considered.

The slide below reveals the cut-off criteria for the different stages.

64 | P a g e

K N O W M E D G E

Other indications for Oxygen therapy in COPD patients are:

PaO2 less than 55 mm Hg or Oxygen saturation less than 88% OR


PaO2 less than 59 mm Hg or Oxygen saturation greater than 88% with evidence of Cor
pulmonale (Right ventricular dysfunction) or secondary erythrocytosis (hematocrit
greater than 55%)

Pearl # 6: IgE and Eosinophil levels help us distinguish ABPA, Hypersensitivity


pneumonitis, and Churg-Strauss syndrome
ABPA Increased IgE levels and increased peripheral eosinophils >10% Rx with steroids

65 | P a g e

K N O W M E D G E

HYPERSENSITIVITY PNEUMONITIS IgE levels and peripheral eosinophils are normal


Remove offending agent
CHURG-STRAUSS SYNDROME IgE levels are normal, peripheral eosinophils >10%
(Clue: asthmatic patient with increase peripheral eosinophils and a foot drop) Management
with steroids
Pearl # 7: Lights criteria will guide you to correctly identifying Pleural Effusions as
either Exudative or Transudative
Broken down into Transudative and Exudative effusion. Remember that for Transudate, all of
the following need to be met. If all are not met, then the patient has an exudative effusion.

66 | P a g e

K N O W M E D G E

Transudate effusions include conditions such as: CHF, Nephrotic syndrome, Cirrhosis,

Hypothyroidism
Exudative effusions include conditions such as: Neoplasm, Infection, RA, SLE,
Esophageal perforation, Pancreatitis, and Dressler Syndrome

Pearl # 8: Bronchoalveolar lavage (BAL) findings can help narrow down, if not nail, the
diagnosis.

Increased Neutrophils think Idiopathic Pulmonary Fibrosis


Increased CD8>CD4 think Hypersensitivity Pneumonitis
Increased CD4>CD8 think Sarcoidosis
Increased Eosinophils think Eosinophilic pneumonia
Positive Silver Methanamine Stain think Pneumocystis Jiroveci in patients with HIV
Inclusion bodies think CMV pneumonia

Once again, the folks who write the Internal Medicine licensing exams dont expect you to have
the depth of knowledge regarding lung conditions that a pulmonologist possesses. However, topics
such as the ones mentioned in the slides and pearls above should assist you with the pulmonary
section of the med school clerkship shelf and ABIM board exams.

67 | P a g e

K N O W M E D G E

5 Rheumatology Pearls
By: Dr. Sunir Kumar
Rheumatology is a subject that comes up daily in clinical practice, and is a favorite on the
ABIM Internal Medicine board exam. According to the ABIM Internal Medicine exam blue print,
Rheumatology / Orthopedics represents 8% of the exam. Here we cover five evidence-based
high-yield pearls that will help you be better prepared for the Internal Medicine ABIM
certification exam!
Pearl # 1: Rheumatoid Arthritis is a systemic, inflammatory and symmetrical condition

Systemic means that it not only involves joints but also will affect different parts of the
body like the lungs, heart, blood vessels, skin, kidneys, and the hematological system.
Inflammatory means that the joints that are affected will be erythematous, warm,
swollen, and tender to touch. Since it is an inflammatory condition, ESR and CRP
(inflammatory markers) will also be elevated.
Symmetrical condition means that both sides of the body will be affected. The main
joints that are affected are the wrist, MCP, and PIP joints.
DIP joint and lower back are usually not affected.
Most specific antibody for RA is anti-CCP.
The most common extra-articular manifestation of RA is subcutaneous nodules.
Poor prognostic factors for RA include:
o Progressive synovitis
o Vasculitis (ulcers of fingers and toes)
o Subcutaneous nodules
o HLA-DR4 marker
o Elevated ESR
o Elevated Rheumatoid Factor
o Erosive lesions on X-ray
Patients with syncope or numbness/tingling in the upper extremities or weakness may
have atlanto-odontoid subluxation. MRI of the cervical spine is the diagnostic test of
choice. Surgical compression is warranted if patient has symptoms or the size is greater
than 8 mm in diameter
68 | P a g e

K N O W M E D G E

Certain medications used in the management of RA are: NSAIDs, Hydroxychloroquine,


Sulfasalazine, Methotrexate with Folic Acid, Leflunomide, Steroids, and anti-TNF alpha
inhibitors
o Hydroxychloroquine frequent eye exams required
o Methotrexate Check CBC and Liver function tests every 6-8 weeks
o Leflunomide Contraindicated in pregnancy. Reverse with cholestyramine X
11 days
o Anti-TNF alpha inhibitors PPD testing needs to be checked before starting
medication. NEVER give two anti-TNF alpha inhibitors concurrently as this
increases the risk of infections. Usually if one is not working, another anti-TNF
alpha inhibitor will work.

Pearl # 2: Osteoarthritis is a chronic, progressively debilitating disease that is noninflammatory and non-systemic

Non-inflammatory means that this condition does not present with erythema or warmth
to a joint but can have swelling. Since it is a non-inflammatory condition, the
inflammatory markers (ESR and CRP) are also normal usually.

Non-systemic means that only joints get affected without compromising the integrity of
the entire body.
Pathophysiology is based on progressive destruction of cartilage that surrounds bone
that leads to bone on bone phenomenon.
Some of the major risk factors for Osteoarthritis include obesity, repetitive use, older
age, and trauma to a joint.
Joint involvement can be mono-articular or asymmetrical, chronic poly-articular.
Major joints that are involved are the hip joint, knee joint, lower back, PIP (Bouchards
nodes), and DIP joints (Heberden nodes). Usually MCP joint is spared.
Mainstay of treatment is non-pharmacological, pharmacological, or surgical.

o Non-pharmacological intervention includes weight reduction and weight


resistance training.
o Pharmacological intervention includes NSAIDs and narcotics.
o Surgical intervention includes steroid injections or knee replacement therapy.

69 | P a g e

K N O W M E D G E

Pearl # 3: Knowing antibodies and their associated conditions are very high yield for
the ABIM board examination

ANA screening test for SLE. Most specific for SLE is anti-Smith, followed by antidsDNA.
Anti-dsDNA is often seen in patients with lupus nephritis.
Anti-histone Drug induced lupus (most common drugs are Procainamide,
Hydralazine, INH, PTU, Minocycline, and Methyl-Dopa)
Anti-CCP most specific antibody for Rheumatoid Arthritis

Anti- SSA (Ro) and Anti SSB (La) commonly seen in Sjogrens Syndrome. NOTE:
Anti-SSA (Ro) can lead to a newborn having complete heart block

Diffuse systemic sclerosis (Scleroderma) anti-SCL 70 (anti-topoisomerase 1)


CREST syndrome anti-centromere
Polymyositis/Dermatomyositis anti Jo-1
Mixed Connective Tissue Disease anti RNP

Pearl # 4: Interpretation of joint effusion plays a critical role in establishing diagnosis

WBC of 200-2000 in joint effusion-non-inflammatory conditions like Osteoarthritis

WBC of 5000-50,000 in joint effusions-inflammatory conditions like RA, gout,


pseudogout, or trauma
WBC >50,000 likely septic arthritis
With gout, inflammatory joint effusion will be seen. Additionally, monosodium urate
crystals will be present and negative birefringence is present
With pseudogout, inflammatory joint effusion will be suspected. Calcium
pyrophosphate crystals will be seen and positive birefringence will be present
With septic arthritis, if patient is less than 40 years of age, the likely causative agent is
Neisseria gonorrhea and treatment is with Ceftriaxone. In patients more than 40 years

of age, likely causative agent is Staph aureus and treatment is with Nafcillin for MSSA
or Vancomycin for MRSA.
NOTE: Never start uric acid lowering agent in an acute gouty attack AND never
discontinue uric acid lowering agent if patient already on a uric acid lowering agent in
an acute attack.

Goal uric acid level to prevent further attacks of gout should be less than 6 mg/dl.
70 | P a g e

K N O W M E D G E

In an acute attack, checking a uric acid level has no diagnostic value.

Pearl # 5: Seronegative Spondyloarthropathies (HLA B27+ and Rh factor negative)

HLA B27 is NEVER used in the diagnosis of the seronegative spondyloarthropathies


Mnemonic to remember the different seronegative spondyloarthropathies is PEARR
o P Psoriatic arthritis
o E Enteropathic arthritis
o A Ankylosing Spondylitis
o R Reactive arthritis
o R Reiter Syndrome causative agent is Chlamydia. Triad of urethritis, uveitis,
and arthritis (Cant pee, cant see, cant climb a tree)

71 | P a g e

K N O W M E D G E

Bonus: How to study for and pass the ABIM board exam
By: Dr. Ravi Bhatia
As the ABIM internal medicine certification exam approached, we received a large number of
emails from our subscribers asking for suggestions on the best way to study for the boards.
The truth is there is no one path to success though there are certainly ways to increase your
likelihood of passing. Regardless of whether you are preparing for board certification or trying
to achieve maintenance of certification (MOC), the best tried and true overall method is to
study early and study often. Below we lay out possible strategies and tactics (in no particular
order) for passing the ABIM board exam:
1. Know the basics of the Internal Medicine board exam
It may seem obvious but a lot of people simply dont review this prior to starting their exam
preparation and instead rely on their ABIM study source of choice to provide the information.

Review the ABIM exam blueprint and understand the topics covered on the exam

A large percentage (33%) of the exam is comprised of Cardiovascular Disease,


Gastroenterology, and Pulmonary Disease

Over 75 percent are based on patient presentations most take place in an outpatient
or emergency department; others are primarily in inpatient settings such as the
intensive care unit or a nursing home.
While its not a big part of the exam, be prepared and expect to interpret some pictorial
information such as electrocardiograms, radiographs, and photomicrographs (e.g.,
blood films, Gram stains, urine sediments).

2. Use the in-training exam as a starting gauge


If you are a resident, the Internal Medicine in-training exam is a good starting point to see
where you stand. Its simply that a barometer of where you stand. It will give you an idea
where you may be weak and where you may be pretty strong. It will also give you an idea of
how you compare with your peers. Dont alter your ABIM study plan simply based on it but it
does give you an early metric of the areas you need to focus on.

72 | P a g e

K N O W M E D G E

3. Get a study guide to prepare for the ABIM exam


Its important to have a good study guide that is tailored for the exam. Some of the more
popular and effective guides weve come across are the MedStudy Internal Medicine Board
Review books and Harrisons Principles of Internal Medicine Board Review.
4. Join a study group
Study groups, if utilized properly, are particularly effective because they allow you to learn
from your colleagues and other exam takers. Oftentimes, people will form study groups with
their colleagues (ideally limited to 3-4 people) at their residency program. Tactics to use in
ABIM study groups may include:

Focus on a new Internal Medicine category by week. For example, focus one week on
cardiology and the next on pulmonary and critical care. The exam can be broken into
a dozen or so categories (see the ABIM exam blueprint). The majority of the
subspecialty questions on the Internal Medicine board exam will focus on cardiology,
gastroenterology, and pulmonary care. However, do not neglect the other areas as the
ABIM wants to ensure that internists have a broad base of medical knowledge.

Test each other with Internal Medicine questions you have written yourself. We are firm
believers in the philosophy that the best way to learn is to teach. If you help others
learn, your knowledge of medical concepts will be greatly strengthened.

We recognize that joining a study group is often not feasible especially for those no longer
in residency programs where everyone is preparing the boards. Fortunately, we live in a digital
age where being part of a study group is much easier. You can connect with colleagues
through Skype, Google Hangout or a number of other channels. One of our favorite
approaches is to remain informed and learn through the power of social media in particular
Twitter. In a previous post, we highlighted excellent Twitter handles to follow for ABIM exam
review as you prepare for certification. Follow @Knowmedge for twice daily questions and our
weekly #ABIMTweetShow hour-long sessions on Sundays. If Twitter is not your cup of tea,
you can also connect with colleagues through the Knowmedge ABIM community on Google+.
Regardless of what approach you decide, studying alongside others preparing for the same
exam is a great motivational tool for success.
73 | P a g e

K N O W M E D G E

5. Get a question bank that fits your personal needs


What is the value of an Internal Medicine question bank? This is a discussion near and dear
to our heart, of course. Question banks have become a popular tool because they bring
together a lot of material in a question format and help create a test taking environment. There
are a lot of question banks to choose from so what should you look for in an ABIM qbank?

High quality ABIM-style questions in a format similar to the exam: The exam is
mostly filled with clinical vignettes and has straightforward questions as well. At a

minimum, your ABIM exam question bank should have both of these types of questions.
Quantity is important but the quality of the questions and explanations is much more
important.
Detailed explanations that review why the incorrect choices were wrong: A
question bank that does not provide you detailed explanations is probably not worth the
money and time spent. As you review questions, you will inevitably get some wrong
your choice of ABIM question bank should detail why your choice is incorrect and the
reasoning behind the correct choice.
Ability to track your personal performance: Your choice of ABIM qbank should be
able to tell you your performance overall and by category. Most not all question
banks provide you a dashboard broken down by category. The Knowmedge question
bank has gone an additional step to break the categories into subcategories as seen
on the ABIM exam blueprint. This allows you to review your strengths and weaknesses
at a granular level. Knowing you are weak at cardiovascular disease is great knowing
you are weak at arrhythmia questions is more valuable.
Add-ons Notes, Lab values, Highlighting: Depending on how you study, these may
be valuable features.

ABIM exam questions straight talk:

No question bank not MKSAP, not Knowmedge, not any knows what will be
on the actual ABIM exam. Based on the ABIM Blueprint, you can make assumptions
on what are the most high-yield areas to study. The point of a question bank is not to
give you the exact questions that will be on the exam it is to hopefully teach you
concepts you may see on the exam and how to reason through what you dont know
immediately.
74 | P a g e

K N O W M E D G E

High-quality ABIM exam review questions can be found in many places


question banks are not the only place. There are study guides, books, and even free
sources. So dont simply base your decision on question bank on the questions. In
addition to the quality of the questions, what truly differentiates one ABIM exam
question bank from another is whether it will truly help you build a broad base of
knowledge and help you retain information for the exam. If you are not comfortable
reading a bunch of text it wont matter how great the questions are. If you are not an
audio-visual learner, the MedStudy or Knowmedge videos wont do anything for you
(As clarity, the Knowmedge qbank contains text and audio-visual explanations for this
exact reason). If you are an old-fashioned learner that prefers printouts
USMLEWorld is definitely not for you those who have used them are well aware their
software will block you from taking print screens or copying of their content. In short
dont follow the herd each one of us learns differently and you need to pick the best
method for you.

6. Consider whether a review course is right for you


There are pros and cons to taking a review course for your ABIM exam prep. The pros are
that it gives you a serious dose of review in a short period of time. It gets you focused if you
werent focused and some courses are absolutely excellent we know some internists are
ardent supporters of some of the professors that teach these courses.
The cons of a review course are that they are expensive (Often over $1,000 plus hotel stay)
and can be inconvenient to travel to and from. Regardless of whether you attend a review
course or not, it cannot replace the pre and post-course study time that is needed. It is
complementary to study time and does not replace it.
7. Review our suggested ABIM test taking strategies
The ABIM exam questions are not intended to trick you they are intended to challenge your
knowledge and ability to bring together your understanding of many different concepts and
topics. Below are some of the tactics you can use as you are practicing questions and/or taking
the actual ABIM exam:

75 | P a g e

K N O W M E D G E

1. For clinical vignettes, read the question (last line) first and then go back and read the
2.
3.
4.
5.
6.

scenario. This way youll know what to look for as you are reading the scenario.
Try to answer the question even before seeing the answer choices.
Pay attention for keywords that can clue you in on an etiology or physical exam.
Watch for key demographic information Geography, ethnicity, gender, age,
occupation.
The ABIM test is not intended to be tricky but we are all human so we miss keywords
sometimes such as least likely pay attention to these.
If you are challenged by a longer clinical vignette, note the key items and develop your
own scenario this may trigger an answer.

7. Most internists weve spoken with say time is generally not an issue but be aware
that it is a timed exam and that you have approximately two minutes per question.
We cannot stress enough the mantra study early and study often. The exam is challenging
but it can be conquered with diligence and proper preparation.
8. Understand and be prepared for ABIM test day

Be prepared and confident. No matter how you have chosen to study, on test day
confidence is critical!
Get a good nights rest last minute cramming and staying up late is only going to
stress you out more.
Get there early dont risk getting caught in traffic. Its much better to be a little early
than be aggravated in traffic.
Take an extra layer of clothing. The last thing you want to do is be uncomfortable and
cold because someone decided to turn on the air conditioner too high.
Test day is long! Be mentally prepared for it. From registration to the optional survey at
the end, the day will be 8-10 hours long (depending on whether you are certifying for
the first time or taking the maintenance of certification exam).
Keep some power snacks with you to take during break time.
Review the ABIM exam day schedule so you know exactly what to expect.

Thats a basic overview of how to study for and pass the ABIM board exam. As mentioned,
there is no secret sauce or method to this you simply need to have a broad base of
knowledge. There is no substitute for studying early and studying often! If you are preparing
76 | P a g e

K N O W M E D G E

for the ABIM Boards, we wish you well were here to help so let us know if you have any
questions! Happy studying!

77 | P a g e

K N O W M E D G E

Bonus: How to study for the Internal Medicine Shelf Exam


By: Dr. Ravi Bhatia
The NBME Internal Medicine Shelf Exam is challenging especially from the standpoint that it
occurs during your internal medicine clerkship a period that you are likely spending a lot of
time at the hospital. This means whatever precious time you have to study, it needs to be
focused on high-yield exam preparation. While there is no one way to study for the NBME
Internal Medicine Shelf Exam, here we present some of the best practices weve picked up
over time. As is the case with any board exam, the best tried and true overall method is to
study early and study often.

1. Take a sneak peek at the shelf exam outline even before your rotation starts, if you
can.

Unbeknownst to many medical students, the NBME publishes an Internal Medicine content
outline of the covered subjects on the Internal Medicine shelf exam. As you review the list of
systems, take a few moments to browse through the review book of your choice (more on this
later) and familiarize yourself with the major diagnoses you can expect to see during your
rotation.

As is the case with many medicine exams, Cardiovascular Disease is the basis for more
questions than any other organ system. A large percentage (35% 50%) of the exam is
comprised of Cardiovascular Disorders, Diseases of the Respiratory System, and Nutritional
& Digestive Disorders.

2. Get a study guidedigital or printto prepare for the NBME exam and your clerkship
Its important to have a good study guide that is tailored for the exam while also preparing you
for the patients youll encounter on the wards. Some of the more popular and effective guides

78 | P a g e

K N O W M E D G E

weve come across that cover both objectives are: Master the Wards Internal Medicine
Clerkship: Survive Clerkship and Ace the Shelf Exam and Step-Up to Medicine

The former is written by Dr. Conrad Fischer who has decades of experience teaching medicine
at all levelsmed school, residency, etcand it shows in this book. His emphasis on clinical
features, diagnostic workup and management of commonly encountered diseases is fairly
comprehensive and yet easy to follow. Most students seem to find that by reading the relevant
sections/chapters of this book related to their patients, they are able to answer just about any
question an attending or resident asks them on rounds. Over the course of the clerkship, that
serves as a huge confidence boost that can translate into a better performance on the shelf
exam.

Step-Up to Medicine is especially strong because it covers diseases in such detail that even
in the absence of another reference, you should be able to confidently learn the material
needed to take care of your patients in the hospital or clinic and also pass the Shelf Exam.
Easy-to-follow, colorful flow charts are an added bonus. If youre asked by your attending to
present a diagnosis related to one of your ward patients, dont be surprised if Step Up is the
first book you find yourself browsing. Itll systematically cover the signs, symptoms, diagnosis,
treatment and potential complications. While I hesitate to say that any resource is a musthave while you are on a medical rotation since there are so many ways to succeed, this book
is the closest you get to a mandatory reference.

Undoubtedly, however, you will come across many of your medical student colleagues
carrying First Aid for the Medicine Clerkship book. I myself used a previous edition of this book
during my Internal Medicine rotation and felt that it didnt go into enough details to lead to a
mastery of the material clinically or for the shelf exam. Even if the shelf exam doesnt ask
minute details, the reference book you choose should provide some context to each disease
rather than concentrating too much on mnemonics, which is what I feel First Aid focused on.
It also wasnt easily applicable to the patients one might expect to see while on rotation.
Perhaps, folks still gravitate to this title because of the fact that First Aid for the USMLE Step
79 | P a g e

K N O W M E D G E

1 an absolutely priceless resourceis fresh in the mind of third year medical students on
their IM rotation, having taken the Step 1 exam just months earlier.

Lastly, since 2000, one book has become more recognizable on the Internal Medicine wards
than any other: Pocket Medicine, which proudly states that it is Prepared by residents and
attending physicians at Massachusetts General Hospital. From a marketing standpoint, the
book is brilliant. The collective knowledge of the worlds premier institution in the pocket of my
white coat? Who can say no to that?

Like the strategy behind the iPhone, each new edition of Pocket Medicine is easy to identify.
When you discover that the latest, latest edition is colored purple, you start to feel that your
green Pocket Medicine book handed down from a recent graduate seems grossly inadequate.
It feels as uncool and antiquated as carrying around a BlackBerry phone. Pocket Medicine
works for some folks; it has to or it wouldnt still be around after a decade and a half. However,
I found the tiny print to be incredibly difficult to navigate. Because the emphasis is on cramming
information into the limited space, the content does not flow nearly as well as Step Up. While
there are ample citations, given that the study cant be clicked, it isnt convenient. To better
view cited material, I would use UpToDate, which your medical center likely offers, at least for
computers on the premises.
3. Thriving on the Internal Medicine rotation doesnt guarantee success on the NBME
Exam but it sure helps!

Your weeks-long rotation in inpatient and outpatient Internal Medicine is not designed to
prepare you for the end of the clerkship NBME shelf exam. Plain and simple. It is intended
instead to familiarize you with the common (and some not-so-common) conditions that
internists can expect to see in practice. By knowing those diagnoses like the back of your
hand, you can better spend your study time reviewing the more esoteric diagnoses you
probably wont come across in the patients on your census.

80 | P a g e

K N O W M E D G E

Treat each and every patient you admit from the emergency room, write a SOAP note on in
the general medicine floors, and see in the exam room of a clinic as an incredible learning
opportunity. Dont forget that as a student you put in early mornings and late nights to study
human pathophysiology, anatomy, genetics, ethics, etc to be given the privilege of seeing live
patients. This is your chance to not only be a part of an actual patient care team but also finally
correlate the tons of medical lectures to a real patient, not a synthesized problem-based
learning (PBL) case.

As a senior resident, I recall often starting my third year medical students with admissions that
on the surface seemed relatively basic: an alcoholic with acute pancreatitis, an obese middleaged man with chest pain after consuming a fatty meal, an 80-year-old female with a 60-packyear smoking history presenting with her third COPD exacerbation of the year. But they were
easy admissions only at the superficial level because it was easy to get fooled into thinking
that identifying the diagnosis was the goal of our trade. In fact, these admissions were chock
full of medical knowledge, provided you successfully opened your mind. Even though the
diagnosis is screaming out at you (often the case with the thorough work-up our Emergency
Medicine colleagues perform and the promptness of imaging reads by our Radiology friends),
maintaining a broad enough differential allows you to be prepared for the next patient who
may have an atypical presentation of a common diagnosis. That, of course, is the type of
patient that one finds presented on the NBME exam.
Ive noticed that with the truncated work-hour schedule in residency, education of residents
and medical students alike has been cut substantially. Even if this means youre not getting
pimped by your attending or senior, read up on each patients complaints. In other words,
that patient with pancreatitis should send you on an exploration of the differential diagnosis
based on the location of the abdominal pain. Even within pancreatitis, use the I GET
SMASHED mnemonic to branch out and learn about each of those topics separately. For
instance, the G stands for Gallstones, which should lead to a review of the diagnosis,
treatment, and complications of cholelithiasis.

81 | P a g e

K N O W M E D G E

Rather than trying to serve as an additional intern and take care of as many patients as
possible (remember medical student doesnt equal workhorse), use the experience of
taking care of a reasonable number of patient to learn about them and their conditions as well
as you can. Ultimately, that will serve you well for developing a solid fund of knowledge and
experience youll be able to apply for years while better preparing you for the NBME exam at
the same time.

4. Join a study group or at least get a study partner

It may seem impossible to find the time on your third-year Internal Medicine clerkship to
coordinate your schedule with other students. Having been in those shoes before, I can tell
you that it can be done. Often, students are given either a Saturday or Sunday off. If you look
around at the beginning of your clerkship orientation, you should be able to find another
student with a similar work schedule.
Theres nothing quite like learning from your colleagues. How do you find a partner who
matches your intelligence? It doesnt matter what their IQ is relative to yours. You simply need
a partner who shares your passion for learning. Even if you come across questions for which
neither you nor your partner have an answer to, a textbook, reliable website (and most likely
Knowmedge) surely will. And if you find that you actually know more than the person you are
studying with, youll be happy to know that nothing reinforces concepts than teaching them to
others. An additional benefit of having a study buddy: A few minutes (not much longer than
that) can be spent debriefing your fellow medical student on the quirkiness of your attending,
idiosyncrasies of your senior attending, and coolness of your intern, etc. Nothing is quite as
soothing as having someone who can relate to your situation.
In the event that youve been stationed in some remote location far, far away from your other
class members, dont despair. Fortunately, we live in a digital age where being part of a study
group is much easier. You can connect with colleagues through Skype, Google Hangout or a
number of other channels. One of our favorite approaches is to remain informed and learn
82 | P a g e

K N O W M E D G E

through the power of social media in particular Twitter. In a previous post, we highlighted
excellent Twitter handles to follow for internal medicine board review. If Twitter is not your cup
of tea, you can also connect with colleagues through the High-Yield Internal Medicine
community on Google+. Regardless of what approach you decide to use, studying alongside
others preparing for the same exam is a great motivational tool for success.

5. Get a question bank that fits your personal needs

What is the value of an Internal Medicine question bank? This is a discussion near and dear
to our heart, of course. Question banks have become a popular tool because they bring
together a lot of material in a question format and help create a test taking environment. There
are a lot of question banks to choose from so what should you look for in an NBME qbank?

High quality NBME-style questions in a format similar to the exam: The exam is
mostly filled with clinical vignettes and has straightforward questions as well. At a
minimum, your NBME exam question bank should have both of these types of
questions. Quantity is important but the quality of the questions and explanations is
much more important.

Detailed explanations that review why the incorrect choices were wrong: A
question bank that does not provide you detailed explanations is probably not worth the
money and time spent. As you review questions, you will inevitably get some wrong
your choice of NBME question bank should detail why your choice is incorrect and the
reasoning behind the correct choice.

Ability to track your personal performance: Your choice of NBME qbank should be
able to tell you your performance overall and by category. Most not all question
banks provide you a dashboard broken down by category. The Knowmedge question
bank has gone an additional step to break the categories into subcategories as seen
on the NBME exam blueprint. This allows you to review your strengths and weaknesses

83 | P a g e

K N O W M E D G E

at a granular level. Knowing you are weak at cardiovascular disease is great knowing
you are weak at arrhythmia questions is more valuable.

Add-ons Notes, Lab values, Highlighting: Depending on how you study, these may
be valuable features. NBME exam questions straight talk:

No question bank not MKSAP for Students, not Knowmedge, not any knows what will be
on the actual NBME exam. However, the NBME blueprint helps to understand the areas that
are emphasized the most. Granted, you still are going to need to study the whole curriculum,
but it can certainly alleviate some of the anxiety when down the stretch, you are unsure of one
of the topics that forms a smaller percentage of the questions. With limited time to study, you
can better choose which high-yield subject areas to study. During the development of
Knowmedges qVault, the entire team focused our energy not on trying to give the exact
questions that will be on the exam. Instead, we look at the sign of an excellent question bank
as teaching important medical concepts that are also useful for the exam.
High-quality NBME exam review questions can be found in many places question banks are
not the only place. There are study guides, books, and even free sources. So dont simply
base your decision on a question bank on the questions. In addition to the quality of the
questions, what truly differentiates one NBME exam question bank from another is whether it
will truly help you build a broad base of knowledge and help you retain information for the
exam. If you are not comfortable reading a bunch of text it wont matter how great the
questions are. If you are not an audio-visual learner, the MedStudy or Knowmedge videos
wont do anything for you (As clarity, the Knowmedge qbank contains text and audio-visual
explanations for this exact reason). If you are an old-fashioned learner that prefers printouts
USMLEWorld is definitely not for you those who have used them are well aware their
software will block you from taking print screens or copying of their content. In short dont
follow the herd each one of us learns differently and you need to pick the best method for
you.
6. Review our suggested NBME test taking strategies
84 | P a g e

K N O W M E D G E

The NBME exam questions are not intended to trick you they are intended to challenge your
knowledge and ability to bring together your understanding of many different concepts and
topics. As mentioned above you will see atypical presentations of common diagnoses or
typical presentations of the uncommon diagnoses. Below are some of the tactics you can use
as you are practicing questions and/or taking the actual NBME exam:

For clinical vignettes, read the question (last line) first and then go back and read the
scenario. This way youll know what to look for as you are reading the scenario.

Try to answer the question even before peeking at the answer choices.

Watch for key demographic information geography, ethnicity, gender, age,


occupation.

The NBME test is not intended to be tricky but we are all human so we miss keywords
sometimes such as least likely pay attention to these. Fortunately, exams have
cut down on including these but you may still come across them.

If you are challenged by a longer clinical vignette, note the key items and develop your
own scenario this may trigger an answer.

Most medical students Ive spoken with say time is generally not an issue 100 questions in
2 hours means 90 seconds per questionbut be sure to maintain the pace recognizing that
its not uncommon to find yourself slowing down towards the end. Get off to a steady start to
save time for the home stretch. We cannot stress enough the mantra study early and study
often. The exam is challenging due to the breadth of Internal Medicine topics but it can be
conquered with diligence and proper preparation.

7. Understand and be prepared for Shelf exam day


Be prepared and confident. No matter how you have chosen to study, on test day confidence
is critical! Get a good nights rest last minute cramming and staying up late is only going to
85 | P a g e

K N O W M E D G E

stress you out more. Get there early dont risk getting caught in traffic. Its much better to be
a little early than be aggravated in traffic.

Take an extra layer of clothing. The last thing you want to do is be uncomfortable and cold
because someone decided to turn on the air conditioner too high.
Thats a basic overview of how to study for and pass the NBME Internal Medicine Board Exam.
As mentioned, there is no secret sauce or method to this you simply need to have a broad
base of knowledge. There is no substitute for studying early and studying often! If you are in
the middle of your Internal Medicine rotation or about to start, we wish you well were here
to help so let us know if you have any questions! Happy learning!

86 | P a g e

K N O W M E D G E

About our Authors


Sunir Kumar, MD Chief Editor
Dr. Kumar is a Board Certified Internist at the Cleveland Clinic. Earlier, he practiced
at UPMC in Pittsburgh, PA as a hospitalist. Dr. Kumar graduated medical school
from St. Matthews University and completed his internal medicine residency at
Mercy Hospital and Medical Center in Chicago, IL.

Ravi Bhatia, MD Associate Editor


Dr. Bhatia is a Board Certified Internist in Fremont, CA. Dr. Bhatia graduated from
the David Geffen School of Medicine at UCLA and completed his internal medicine
residency at Allegheny General Hospital in Pittsburgh, PA.

Salim R. Rezaie, MD
Dr. Rezaie is currently an attending on the faculty of UTHSCSA in San Antonio,
TX. He completed his medical school training at Texas A&M Health Science
Center, and followed that up with a combined Emergency Medicine/Internal
Medicine residency at East Carolina University in Greenville, NC.

Sheila Krishna, MD
Dr. Sheila Krishna is currently Chief Resident of the Department of Dermatology
at the Medical College of Virginia/Virginia Commonwealth University Health
System. She is interested in complex medical dermatology and general
dermatology.

Ruchi Bhatia, MD
Dr. Bhatia is currently in her final year of Internal Medicine residency at St. Louis
University. She obtained her medical degree from Northeastern Ohio Medical
University in Rootstown, Ohio. Dr. Bhatia will be pursuing a fellowship in
Gastroenterology and is specifically interested in the area of Liver Disease.

87 | P a g e

K N O W M E D G E

Check out additional study resources at www.knowmedge.com

900+ questions
Animated video explanations
Printable explanations
Notes, highlights, flags
Interactive dashboard
Personalized study suggestions
Medical & diagnostic categorization
14 day money-back guarantee

88 | P a g e

You might also like